Sie sind auf Seite 1von 39

Editor-in-Chief: Joseph A.

Zenel, Sioux Falls, SD


Associate Editor: Hugh D. Allen, Houston, TX.
Editor, In Brief: Henry M. Adam, Bronx, NY
Consulting Editor, In Brief: Janet Serwint, Baltimore, MD
Editor, Index of Suspicion: Deepak M. Kamat, Detroit, MI
Editor, CME: Paula Algranati, Longmeadow, MA
Editor Emeritus: Lawrence F. Nazarian, Rochester, NY
contents
Founding Editor: Robert J. Haggerty, Canandaigua, NY
Managing Editor: Luann Zanzola Pediatrics in Review® Vol.34 No.12 December 2013
Editorial Associate: Sara Strand
Medical Copyediting: Laura King aap.org/education
Editorial Board
Denise Bratcher, Kansas City, MO Michael Macknin, Cleveland, OH
Brian Carter, Nashville, TN Susan Massengill, Charlotte, NC
Deborah M. Consolini, Wilmington, DE Jennifer Miller, Gainesville, FL
Stephen E. Dolgin, New Hyde Park, NY
B. Anne Eberhard, New Hyde Park, NY
Philip Fischer, Rochester, MN
Carrie A. Phillipi, Portland, OR
Peter Pizzutillo, Philadelphia, PA
Mobeen Rathore, Jacksonville, FL
Articles
Lynn Garfunkel, Rochester, NY E. Steve Roach, Columbus, OH
Rani Gereige, Miami, FL Sarah E. Shea, Halifax, Nova Scotia
Joseph Gigante, Nashville, TN Andrew Sirotnak, Denver, CO
Type 2 Diabetes Mellitus in Children
Nupur Gupta, Boston, MA
Jacob Hen, Bridgeport, CT
Jeffrey D. Hord, Akron, OH
Alfred Tenore, Udine, Italy
Miriam Weinstein, Toronto, ON
541 and Adolescents
Kavitha Dileepan, M. Max Feldt
Publisher: American Academy of Pediatrics
Michael J. Held, Director, Division of Scholarly Journals and Professional Periodicals
Pediatrics in Review® Managing Feeding Problems and Feeding
(ISSN 0191-9601) is owned and controlled by the American Academy of
Pediatrics. It is published monthly by the American Academy of Pediatrics,
141 Northwest Point Blvd., Elk Grove Village, IL 60007-1098.
Statements and opinions expressed in Pediatrics in Review® are those of the
549 Disorders
James A. Phalen
authors and not necessarily those of the American Academy of Pediatrics or
its Committees. Recommendations included in this publication do not indicate
an exclusive course of treatment or serve as a standard of medical care.
Subscription price for 2014 for print and online/online only: AAP/CPS
Member $194/$148; AAP National Affiliate Member $153/$104; Nonmember
$243/$188; Allied Health or In-training $180/$122.
Institutions call for pricing (866-843-2271). For overseas delivery, add $120.
558 Human Metapneumovirus
Jennifer E. Schuster, John V. Williams
Current single issue price is $10 domestic, $12 international. Replacement
issues must be claimed within 6 months from the date of issue and are limited
to three per calendar year. Periodicals postage paid at ARLINGTON
HEIGHTS, ILLINOIS and at additional mailing offices.
© AMERICAN ACADEMY OF PEDIATRICS, 2013. All rights reserved.
Printed in USA. No part may be duplicated or reproduced without permission
565 Corrections
of the American Academy of Pediatrics.
POSTMASTER: Send address changes to PEDIATRICS IN REVIEW®,
American Academy of Pediatrics Customer Service Center, 141 Northwest
Index of Suspicion
Point Blvd., Elk Grove Village, IL 60007-1098. Case 1: Swelling Behind the Knee in a 15-Year-Old Boy
Pediatrics in Review
Print Issue Editorial Board Disclosures Case 2: Fatigue, Weight Loss, Fever, and Atypical Skin
The American Academy of Pediatrics (AAP) Policy on Disclosure of Financial
Relationships and Resolution of Conflicts of Interest for AAP CME Activities is Lesion in an Adolescent Boy
designed to ensure quality, objective, balanced, and scientifically rigorous AAP CME
activities by identifying and resolving all potential conflicts of interest before the Case 3: New-Onset Murmur and Signs of Shock in a
confirmation of service of those in a position to influence and/or control CME content.
All individuals in a position to influence and/or control the content of AAP CME
activities are required to disclose to the AAP and subsequently to learners that the
individual either has no relevant financial relationships or any financial relationships with
566 15-Year-Old Girl
Case 1: Senthilkumar Sankararaman, Kamakshya Patra,
the manufacturer(s) of any commercial product(s) and/or provider(s) of commercial
services discussed in CME activities. Commercial interest is defined as any entity Wanda Wells
producing, marketing, reselling or distributing health-care goods or services consumed
by, or used on, patients. Case 2: Tania Sarker, Brenda Kitchen, Rama Jasty
Each of the editorial board members, reviewers, question writers, PREP Coordinating
Committee members and staff has disclosed, if applicable, that the CME content he/ Case 3: Natalia Benza, Shashi Sahai
she edits/writes/reviews may include discussion/reference to generic pharmaceuticals,
off-label pharmaceutical use, investigational therapies, brand names, and manufacturers.
None of the editors, board members, reviewers, question writers, PREP Coordinating
Committee members, or staff has any relevant financial relationships to disclose, unless
noted below. The AAP has taken steps to resolve any potential conflicts of interest.
In Brief
Disclosures
• Athos Bousvaros, MD, MPH, FAAP, disclosed that he has research grants from Merck
and UCB; and that he is a paid consultant and on the speaker bureau for Millennium.
• Lynn Garfunkel, MD, FAAP, disclosed that her family member is an employee
of Philips Healthcare.
573 Down Syndrome
Wendy L. Hobson-Rohrer, Lisa Samson-Fang
• Blaise Nemeth, MD, MS, FAAP, has disclosed he has an unrestricted
educational grant for fellowship from Biomet.
• Richard Sills, MD, FAAP, disclosed that he receives a research grant from Novartis.
• Miriam Weinstein, MD, disclosed that she is a consultant for Pediapharm
and SunCorp. She also has received honoraria from Dermik and Leo Pharma.
The journal extends special thanks to the following question writers who
Online-only Article
contributed to this issue:
• H. Stephen Williams, MD
• O.J. Sahler, MD
Abstract appears on page 575.

Visual Diagnosis: An 11-Month-Old With


e47 Nausea, Vomiting, and an Abdominal Mass
Laura A. Whittington, David C. Stevens, Sarah A. Jones,
Julie M. Mayo
Continuing Medical Education Live Activities

Pediatrics in Review® is supported, in part, through an


Scholarly Journals and Conferences and Exhibits
educational grant from Abbott Nutrition, a division of
Abbott Laboratories, Inc. Professional Periodicals Employment Resources
Pediatrics in Review offers 36 CME articles per year. A maximum of one AMA PRA Category 1 CreditTM is earned after

Online Resources for


achieving a 60% score on each designated quiz.
Cover: The artwork on the cover of this
Physician Reentry
CME statements:
month’s issue is by one of the winners The American Academy of Pediatrics (AAP) is accredited by the Accreditation Council for Continuing Medical Education

Professional Growth
(ACCME) to provide continuing medical education for physicians.
of our 2012 Cover Art Contest, 8-year- The AAP designates this journal-based CME activity for a maximum of 1.00 AMA PRA Category 1 CreditTM. Physicians should
old Achal G. of Marshfield, WI. Achal’s claim only the credit commensurate with the extent of their participation in the activity.

pediatrician is Dr Stephen Caselton. This activity is acceptable for a maximum of 1.00 AAP credit. These credits can be applied toward the AAP CME/CPD* Award
available to Fellows and Candidate Members of the AAP.
The American Academy of Physician Assistants accepts certificates of participation for educational activities certified for AMA PRA
Category 1 CreditsTM from organizations accredited by ACCME. Physician assistants may claim a maximum of 1.00 hour of Category
1 credit for completing this program.
This program is accredited for 1.00 NAPNAP CE contact hour; pharmacology (Rx) and psychopharmacology contact hours
to be determined per the National Association of Pediatric Nurse Practitioners (NAPNAP) Continuing Education Guidelines.
It has been established that each CME activity will take the learner approximately 1 hour to complete.
Answer key appears on page 574. *Continuing Professional Development
How to complete this activity:
Pediatrics in Review can be accessed and reviewed in print or online at http://pedsinreview.aappublications.org. Learners can
claim credit monthly online upon completion of each CME article. The deadline for completing this activity is December 31,
2015. Credit will be recorded in the year in which it is submitted. It is estimated that it will take approximately 1 hour to com-
plete each CME article. This activity is not considered to have been completed until the learner documents participation in that
activity to the provider via online submission of answers. Course evaluations are online.
Article endocrinology

Type 2 Diabetes Mellitus in Children and


Adolescents
Kavitha Dileepan, MD, MPH*
Educational Gaps
M. Max Feldt, DO†
The growing pandemic of childhood obesity has led to marked increases in the incidence
and prevalence of type 2 diabetes mellitus (DM) and has further complicated the differ-
Author Disclosure entiation between type 2 and type 1 DM because more children with type 1 DM are over-
Drs Dileepan and Feldt weight at time of diagnosis. In addition, numerous studies have demonstrated b-cell
have disclosed no autoimmunity in children with type 2 DM. (1)
financial relationships
relevant to this article.
Objectives After completing this article, readers should be able to:
This commentary does
contain a discussion of 1. Differentiate between and understand the treatment of type 2 diabetes mellitus (DM)
an unapproved/ and type 1 DM by recognizing underlying pathophysiologic characteristics, clinical
investigative use of features, and laboratory markers.
a commercial product/ 2. Recognize the difficulty in distinguishing between type 2 and type 1 DM because of
device. the increasing prevalence of childhood obesity.
3. Understand the increased prevalence of type 2 DM in children and adolescents,
especially among certain racial/ethnic groups.
4. Recognize risk factors, appropriate screening, and diagnosis of type 2 DM, and
understand the appropriate treatment for type 2 DM in the pediatric population.
5. Recognize the comorbidities and complications associated with type 2 DM in children
and adolescents and understand that these may be present at the time of diagnosis.

Introduction
Type 2 diabetes mellitus (DM), historically considered a serious chronic medical condition only
for older individuals, now has an increased prevalence in children and adolescents. The estimated
overall incidence of type 2 DM is 22 cases per 100,000 youth or approximately 3600 youth
diagnosed with the condition each year. (2) From a public health perspective, DM is the seventh
leading cause of death in the United States, a figure that is likely underestimated. (3) The total
cost to treat DM in both adults and children is approximately $174 billion per year, and medical
expenses for individuals with diagnosed DM are 2.3 times higher than for those without DM.
(3) The clinical and financial burdens of DM are increased by the complications and comorbid-
ities of the disease. Because complications of DM develop and worsen during the disease, (2) it is
important to effectively recognize and manage type 2 DM early
when it is diagnosed during childhood and adolescence.
Abbreviations
ADA: American Diabetes Association Definition
DM: diabetes mellitus DM represents a group of endocrine disorders characterized by
FDA: Food and Drug Administration hyperglycemia caused by defective insulin secretion, defective in-
GAD: glutamic decarboxylase sulin action, or both. (4) The original division into 2 types was
HbA1c: hemoglobin A1c based on age at presentation and dependence on insulin. Now
OGTT: oral glucose tolerance test categories of DM (Table 1) are differentiated by their known
TODAY: Treatment Options for Type 2 Diabetes in underlying pathophysiologic characteristics. All forms of DM ul-
Adolescents and Youth timately lead to hyperglycemia, although there may be overlap
in the fundamental pathologic processes in each patient.

*Department of Pediatrics, University of Missouri Kansas City School of Medicine, Children’s Mercy Hospital Kansas City, MO.
Dr Dileepan was a pediatric endocrinology fellow at Children’s Mercy Hospital, Kansas City, MO, at the time this article was authored.

Department of Pediatrics, University of Missouri Kansas City School of Medicine, Children’s Mercy Hospital, Kansas City, MO.

Pediatrics in Review Vol.34 No.12 December 2013 541


endocrinology diabetes mellitus

Type 2 DM disproportionately affects racial and ethnic


Classification of Diabetes
Table 1. minorities. American Indian youth have the highest inci-
dence at 174 per 100,000. Black youth also have a partic-
Mellitus (DM) Causes ularly high incidence of 105 per 100,000 compared with
Type 1 DM: b-cell destruction, typically leading to
19 per 100,000 in non-Hispanic whites. (2)
absolute deficiency in insulin secretion Epidemiologic studies report that children born to
Type 1a: immune mediated, characterized by positive mothers with gestational diabetes are at greater risk of de-
circulating autoantibodies veloping type 2 DM. (8) Breastfeeding appears to have
Type 1b: idiopathic protective effects against the development of type 2
Type 2 DM: clinical spectrum from predominant insulin
resistance with relative insulin deficiency to
DM. (9)
a predominant defect in insulin secretion with
accompanying insulin resistance
Gestational DM Etiology
Other types of DM The cause of type 2 DM is multifactorial, but the high
Genetic defects of b-cell function (ie, monogenic concordance rate among monozygotic twins (1) and
diabetes of youth) the frequent association with a family history of DM
Genetic defects in insulin action (ie, leprechaunism,
Rabson-Mendenhall syndrome, and lipoatrophic DM)
(10) suggest a genetic component. Between 74% and
Diseases of exocrine pancreas or pancreatic 100% of patients with type 2 DM have a first- or
destruction (ie, cystic fibrosis, pancreatitis, trauma, second-degree relative with the disease, in contrast to
and hemochromatosis) only 5% of patients with type 1 DM with a family history
Endocrinopathies (ie, Cushing syndrome, acromegaly, of type 1 DM. (6) Recent studies have identified multiple
glucagonoma, hyperthyroidism, and
pheochromocytoma) genetic loci that are associated with higher risk of type 2
Drug induced (ie, glucocorticoids, diazoxide, DM. For example, polymorphisms in the TCF7L2 gene
b-adrenergic agonists, and asparaginase) result in impaired insulin secretion and defective insulin
Infections (ie, cytomegalovirus and congenital rubella) processing, which confer a 1.4 times increased risk of type
Other uncommon forms of immune-mediated DM 2 DM. (1) More genetic markers are being identified
(ie, stiff man syndrome and anti-insulin receptor
antibodies)
with improvements in genetic testing.
Genetic syndromes associated with DM (ie, Down
syndrome, Turner syndrome, and Klinefelter syndrome) Pathogenesis
Glucose Metabolism and Insulin Production
Glucose metabolism is tightly regulated by several pro-
cesses, including sensing of glucose concentration, insulin
Epidemiology synthesis, and secretion by pancreatic b-cells; suppression
Recent epidemiologic studies have demonstrated that of hepatic glucose output; and insulin action on stimu-
more than 20% of new cases of DM in children and ado- lated glucose uptake by the liver, intestines, and skeletal
lescents are due to type 2 DM. (5) The incidence of type muscle. Hyperglycemia can result from derangements in
2 DM in children has increased in part because of the ep- any of these processes.
idemic of childhood obesity and the associated insulin Typically, type 1 DM and type 2 DM are conceptual-
resistance, although the actual incidence of type 2 DM ized on a spectrum. Type 1 DM results from immune-
in children is likely higher than reported because of mediated destruction of b-cells, leading to insulinopenia.
underdiagnosis. Type 2 DM is the result of obesity-mediated insulin re-
The incidence of type 2 DM increases with age. The sistance and non–immune-mediated deficiency in insulin
SEARCH for Diabetes in Youth study demonstrated that secretion.
the incidence of type 2 DM in children age 10 to 19 years The pathogenesis of type 2 DM is complex and in-
was 42 per 100,000 youth compared with 1 in 100,000 volves interactions between genetic and environmental
youth among children age 0 to 9 years. (2) The peak age factors. The core defect is varying degrees of insulin re-
at onset of type 2 DM in children coincides with pubertal sistance and subsequent progressive insulinopenia. Other
timing because the mean age at diagnosis is 12 to 16 factors associated with obesity, such as elevated plasma
years. (6) Females have a higher incidence of type 2 free fatty acid concentrations and increased inflamma-
DM than males, (5) likely because girls are more insulin tory markers, further inhibit b-cell insulin production
resistant and carry more subcutaneous fat than boys. (7) and insulin-mediated glucose uptake. This leads to

542 Pediatrics in Review Vol.34 No.12 December 2013


endocrinology diabetes mellitus

a cycle of worsening hyperglycemia and further meta- having type 2 DM who were antibody positive were more
bolic derangement. likely to be white and male and had a lower body mass
index than antibody-negative patients. (16) A significant
Role of Obesity controversy exists regarding the classification of antibody-
A theory called the accelerator hypothesis suggests that positive type 2 DM, with many authors maintaining that
obesity and weight gain contribute significantly to b-cell antibody-positive type 2 DM should be considered early
stress and confer earlier onset of all types of DM. Obesity type 1 DM and treated as such.
is increasingly being accepted as a contributor to b-cell
failure in genetically susceptible children, and increasing Clinical Signs and Symptoms
evidence suggests the influence of obesity in abnormal The presenting symptoms of type 1 and type 2 DM can
immune modulation. (1) be similar and include polyuria, polydipsia, and polypha-
gia. Weight loss can be present in both types of DM.
Lifestyle Contributions Clinical signs to suggest type 2 DM include overweight
During the past 30 years Americans have increased their body habitus, with more than 85% of children with type 2
total caloric intake by an additional 300 kcal/d. (11) DM considered overweight or obese at the time of diag-
Consumption of juices and sugar-sweetened beverages nosis. (6) Acanthosis nigricans, a darkened, thick, velvety
is a major source of these additional calories in the diet appearance to the skin found typically in folds or creases,
of children and adolescents (12) and is strongly associ- is present in 90% of patients with type 2 DM and can be
ated with an increased risk of obesity and type 2 DM. the most easily visible clinical indicator of insulin resis-
(7) In addition, fewer children and adolescents are par- tance. (7) The frequency of acanthosis nigricans in obese
ticipating in recommended levels of physical activity. adolescents or hyperinsulinemic children varies consider-
ably by ethnicity. Up to 90% of obese or hyperinsulinemic
In Utero Factors children in Native American populations had acanthosis
The association between lower birth weight and type 2 nigricans, whereas it was present in less than 5% of
DM suggests that in utero programming may increase non-Hispanic white counterparts. (7) Clinicians can look
the risk of type 2 DM. (13) A thrifty-phenotype hypothesis for acanthosis nigricans in the nape of the neck, axilla,
suggests that poor fetal nutrition produces a postnatal groin, and over flexor surfaces. The presence of ketoaci-
metabolism that is adapted to poor but not plentiful nu- dosis, normally found in patients with type 1 DM, does
trition. This programming contributes to insulin resis- not rule out type 2 DM. Some reports indicate that up to
tance and can predispose the development of type 2 25% of children and adolescents with type 2 DM present
DM in the context of excess nutrition and obesity. (14) with diabetic ketoacidosis. (17)
Type 2 DM generally has a more insidious onset than
type 1 DM, and many patients may be asymptomatic at
Clinical Aspects presentation. However, because of the potentially long-
Classification of DM standing hyperglycemia, patients may already have evi-
Because up to 24% of children and adolescents with type dence of microvascular and macrovascular complications
1 DM are overweight at diagnosis, (6) differentiating be- at the time of diagnosis. (10)
tween type 2 DM and type 1 DM has become more dif- Overall, the clinical distinction between type 1 DM
ficult. (15) Further complicating the clinical delineation is and type 2 DM is increasingly obscured, especially with
the presence or absence of autoimmunity. The SEARCH the increasing obesity pandemic. Clinicians must weigh
for Diabetes in Youth Study measured the presence of the evidence to support their diagnosis and consider
glutamic decarboxylase (GAD) antibodies among dia- the potential outcomes of misclassification. In the case
betic patients. Positive GAD antibodies were found in of significant hyperglycemia, diabetic ketoacidosis,
21% of patients with type 2 DM older than 10 years. and/or positive antibodies, it may be prudent to treat pa-
(5) The Treatment Options for Type 2 DM in Adoles- tients as having type 1 DM and wean insulin therapy if the
cents and Youth (TODAY) study, a multicenter clinical future clinical course dictates.
trial, evaluated the presence of GAD and insulinoma-
associated protein 2 antibodies. Of patients with diag- Diagnostic Approach
nosed type 2 DM, 9.8% were antibody positive, 5.9% The criteria for diagnosis of DM were based on data to
were positive for a single antibody, and 3.9% were posi- delineate risk for the development of retinopathy, a mi-
tive for both antibodies. Those patients diagnosed as crovascular complication of DM, and are included in

Pediatrics in Review Vol.34 No.12 December 2013 543


endocrinology diabetes mellitus

Table 2. (10) In 2009, an international expert committee


convened and added an additional criterion for diagnosis, Indications to Test for
Table 3.
a hemoglobin A1c (HbA1c) level greater than 6.5%
(0.07). (10) Type 2 Diabetes Mellitus (DM)
Indications to test for type 2 DM according to the Overweight (body mass index >85th percentile, weight
Type 2 DM Consensus Panel are given in Table 3. Test- for height >85th percentile, or weight >120th
ing should begin at age 10 years or at the age of pubertal percentile for ideal height)
onset, whichever comes first, and should be repeated ev- WITH any 2 of the following risk factors:
ery 3 years. (18) The preferred method is to measure Family history of type 2 DM (first- or second-degree
relatives)
a fasting plasma glucose level, but a 2-hour plasma glu- Race/ethnicity (American Indian, African American,
cose level measured during an oral glucose tolerance test Hispanic/Latino, or Asian/Pacific Islander)
(OGTT) can be an alternative. (6) Signs of insulin resistance (acanthosis nigricans)
Comorbidities of insulin resistance (hypertension,
dyslipidemia, small for gestational age birthweight,
Management cardiovascular disease, hypervirilization,
Ongoing Clinical Management steatohepatitis, and polycystic ovarian syndrome)
The American Diabetes Association (ADA) recommends Modified by permission from American Diabetes Association. Type 2
that treatment of all children with DM should include diabetes in children and adolescents. Diabetes Care. 2000;23(3):381–
389.
routine follow-up every 3 months with a diabetes care
team. (18) This team should include nutritional, psycho-
logical, and educational support and a medical professional
experienced with DM care. The patient’s self-management and symptoms of sleep apnea should occur at diagnosis
involves monitoring of blood glucose level, medication and annually. (19) Coordination between the diabetes
compliance, attention to dietary intake, and physical ac- care team and the primary care practitioner ensures
tivity. Psychosocial considerations and medical compli- a complete medical home for the child or adolescent.
ance must also be addressed to ensure optimal success Treatment should be guided by the acuity of the clinical
with therapy. The HbA1c level should be measured ev- presentation. If a patient is acutely ketotic, dehydrated, or
ery 3 months during outpatient visits, and the goal acidotic, intravenous hydration and insulin administration
HbA1c should be less than 7%. (18)(19)(20) Assessment with inpatient admission are warranted. If the presentation
of lipids, liver function tests, microalbuminuria, and signs is less acute, subspecialty referral, outpatient education,
and use of oral medications can be initiated. Once the pa-
tient is clinically stable, treatment of type 2 DM is dually
focused on weight management and minimizing compli-
cations associated with hyperglycemia.
Criteria to Diagnose
Table 2.

Diabetes Mellitus Lifestyle Modification


Lifestyle modification is an essential component of the
Fasting (>8 hours) blood glucose level ‡126 mg/dL management of type 2 DM and includes an emphasis
(7 mmol/L)
OR on proper diet and exercise to maintain a healthy weight
2-Hour plasma glucose level ‡200 mg/dL (11.1 mmol/L) while preserving linear growth. For optimal management,
during oral glucose tolerance test with 1.75 g/kg lifestyle modification should be centered around the family
(maximum, 75 g/kg) of glucose unit and not strictly on individual patients. (20)
OR
Hemoglobin A1c level ‡6.5% standardized to Diabetes
Control and Complications Trial assay Pharmacologic Management
OR Pharmacologic therapy addresses various aspects of the
Random plasma glucose level ‡200 mg/dL (11.1 mmol/L) pathogenesis of type 2 DM (Figure) by reducing insulin
WITH resistance, increasing insulin secretion, slowing postpran-
signs and symptoms of DM (ie, polyuria, polydipsia, and dial glucose absorption, or supplementing inadequate se-
unintentional weight loss)
cretion of insulin. Metformin, a biguanide, is the only US
Modified by permission from American Diabetes Association. Type 2 Food and Drug Administration (FDA)–approved oral
diabetes in children and adolescents. Diabetes Care. 2000;23(3):381–
389. medication for treatment of type 2 DM in children older
than 10 years and is considered first-line therapy in

544 Pediatrics in Review Vol.34 No.12 December 2013


endocrinology diabetes mellitus

nonacute presentations. The mechanism of action is to in the upper small intestine. They can lower HbA1c levels
decrease hepatic glucose production and enhance insulin- by 0.5% to 1%, and the major adverse effects are gastroin-
mediated glucose uptake in muscle and adipose cells. Ad- testinal intolerance and flatulence. Incretins (exenatide) are
verse effects include transient abdominal pain, diarrhea, and designed to increase postprandial insulin secretion. Exen-
nausea, although it is generally well tolerated. Metformin atide is administered as a twice-daily injection. Adverse ef-
should not be given to patients with renal insufficiency, fects include nausea, vomiting, diarrhea, dyspepsia,
liver disease, or cardiac or respiratory insufficiency. Patients jitteriness, dizziness, headaches, and hypoglycemia, espe-
should be warned to discontinue metformin therapy before cially if given with a sulfonylurea. Thiazolidinediones (ro-
receiving intravenous contrast for radiographic studies be- siglitazone and pioglitazone) increase insulin sensitivity in
cause of the increased risk of lactic acidosis. In adults there muscle, adipose tissue, and the liver. In isolation, they can
is evidence that metformin can normalize blood glucose reduce HbA1c levels by 0.5% to 1.3%. Results from the
levels, decrease cholesterol levels, and reduce hypertension. TODAY study demonstrated that metformin in combi-
Metformin can also be used to normalize ovulatory abnor- nation with rosiglitazone was more successful than met-
malities in patients with polycystic ovarian syndrome. It is formin alone or metformin with lifestyle modification in
available in a liquid formulation and as an extended-release preventing an increase in HbA1c levels above 8% (0.08).
formulation, which can aid compliance and potentially re- (20) There is some evidence that this combination may
duce adverse effects. The recommended starting dose for improve lipid profiles by lowering triglyceride levels
metformin is 500 mg given orally once daily, with a maxi- and increasing high-density lipoprotein levels. (21) Ad-
mum dose of 2000 mg per day. verse effects include edema, weight gain, and anemia
Insulin is used to attain early normalization of glyce- and may infer additional cardiac risk (Figure).
mic control, especially in patients who are acutely ill or
have significant hyperglycemia, and insulin therapy Prognosis
should be started in all patients who present with diabetic The risk of DM-related complications is directly related
ketoacidosis. Recent American Academy of Pediatrics to the duration of disease. Prompt diagnosis and appro-
clinical practice guidelines on the management of newly priate therapy are paramount in reducing this risk. Be-
diagnosed type 2 DM recommend that insulin therapy be cause of its insidious onset, many patients with type 2
initiated in patients who have a random blood glucose DM have evidence of complications at the time of diag-
level greater than 250 mg/dL (13.9 mmol/L) or whose nosis. Among a sample of 100 Pima Indians with type 2
HbA1c level is greater than 9%.
(20) Once the diagnosis of type 2
DM is determined, insulin therapy
can often be reduced as metformin
therapy is initiated. Adverse effects
of insulin can include weight gain
from its anabolic effect on metabo-
lism. Hypoglycemia, a potential ad-
verse effect, is not as common among
patients with type 2 DM.
Other therapies, although not
FDA approved for patients younger
than 18 years, can be used by the di-
abetes management team to improve
glycemic control. Sulfonylureas (gly-
buride, glipizide, and glimepiride)
directly increase insulin secretion,
so they are most useful when there
is residual b-cell function. Major ad-
verse effects include hypoglycemia
and weight gain. Glucosidase inhib-
itors (acarbose and miglitol) re- Figure. Pathogenesis and therapeutic targets.
duce absorption of carbohydrates DM[diabetes mellitus.

Pediatrics in Review Vol.34 No.12 December 2013 545


endocrinology diabetes mellitus

DM, 18% had hypertension, 7% had dyslipidemia, and


22% had microalbuminuria at the time of diagnosis. • On the basis of consensus, diagnosis of type 2 DM can
be confirmed by an elevated fasting blood glucose
(22) Microvascular complications of type 2 DM include level greater than 126 mg/dl (7.0 mmol/L), an
nephropathy, retinopathy, and neuropathy. Macrovascu- elevated 2-hour plasma glucose greater than 200 mg/
lar complications include hypertension and hyperlipid- dL (11.1 mmol/L) on an oral glucose tolerance test, an
emia, which can lead to cardiovascular disease. The elevated random blood glucose greater than 200 mg/
ADA recommends that patients with type 2 DM have dL (11.1 mmol/L), or a hemoglobin A1c level greater
than 6.5% with suggestive symptoms. (10)
blood pressure measurements performed at every routine • According to strong research evidence and consensus,
diabetes visit. (18) In the UK Prospective Diabetes once the diagnosis has been made, treatment should
Study, hypertension was found to be a more significant be based on the acuity of presentation and should
predictor of cardiovascular disease than blood glucose focus on lifestyle modification and on normalizing
control. This study also found a 25% reduction in the risk hyperglycemia to minimize complications. Metformin
is currently first-line treatment for type 2 DM in
of microvascular complications when the average HbA1c children and adolescents older than age 10 years who
level decreased from 7.9% to 7.0%. (23) present nonacutely. (18) (19)
Patients should be screened for retinopathy with a di- • Strong research evidence and consensus demonstrate
lated eye examination near the time of diagnosis and then that because type 2 DM has an insidious onset,
yearly afterward. A lipid profile should be performed microvascular and macrovascular complications can
be present at the time of diagnosis. Patients should be
shortly after diagnosis, once glycemic control is attained, screened for the presence of complications when the
and should be performed annually thereafter. Urine mi- diagnosis of type 2 DM is made and in follow-up. (6)
croalbumin should be measured at diagnosis to assess for (10)
early nephropathy and followed up yearly. (18)
Primary public health prevention efforts should target
the general population to limit the prevalence of obesity. ACKNOWLEDGMENTS. The authors wish to acknowl-
Secondary prevention should focus on screening those edge Babalola Faseru, MD, MPH, assistant professor,
children who are at high risk for the development of type Department of Preventive Medicine and Public Health,
2 DM (Table 2). Tertiary prevention in patients with type University of Kansas School of Medicine, and Naim Mitre,
2 DM should address reduction of complications. MD, Naim Mitre, MD, medical director of clinical services,
assistant fellowship program director, assistant professor of
pediatrics, Division of Endocrine/Diabetes, Department
of Pediatrics, University of Missouri-Kansas City/Children’s
Summary Mercy Hospital.

• On the basis of strong research evidence and Suggested Reading


consensus, type 1 diabetes mellitus (DM) remains the Expert Panel on Integrated Guidelines for Cardiovascular Health
most common form of DM in children and and Risk Reduction in Children and Adolescents; National Heart,
adolescents. The incidence of type 2 DM in the Lung, and Blood Institute. Expert panel on integrated guidelines
pediatric population is rapidly increasing because of for cardiovascular health and risk reduction in children and
the obesity epidemic, and minority groups are adolescents: summary report. Pediatrics. 2011;128(suppl 5):S213–
disproportionately affected. (2) (10) (19) S256
• On the basis of some research evidence and consensus, Hannon TS, Rao G, Arslanian SA. Childhood obesity and type 2
it can be challenging to initially differentiate between diabetes mellitus. Pediatrics. 2005;116(2):473–480
type 2 DM and type 1 DM clinically because of the
increased prevalence of obesity, the complex interplay
of autoimmunity and obesity, and common symptoms References
at presentation. (1) (10) (19) 1. Badaru A, Pihoker C. Type 2 diabetes in childhood: clinical
• Significant evidence and consensus support a genetic characteristics and role of b-cell autoimmunity. Curr Diab Rep.
basis for the development of type 2 DM in children. 2012;12(1):75–81
• Physicians should routinely screen at risk children 2. Liese AD, D’Agostino RB Jr, Hamman RF, et al; SEARCH for
older than age 10 years for DM. Screening criteria Diabetes in Youth Study Group. The burden of diabetes mellitus
include obesity, a family history of type 2 DM, among US youth: prevalence estimates from the SEARCH for
a minority racial or ethnic background, acanthosis Diabetes in Youth Study. Pediatrics. 2006;118(4):1510–1518
nigricans, or other diseases associated with insulin 3. Centers for Disease Control and Prevention. National diabetes fact
resistance, including polycystic ovary syndrome, sheet 2011: national estimates and general information on diabetes
hypertension, or dyslipidemia. (1) (10) (18) (19) and prediabetes in the United States. Available at: http://www.cdc.
gov/diabetes/pubs/factsheet11.htm. Accessed October 16,2012

546 Pediatrics in Review Vol.34 No.12 December 2013


endocrinology diabetes mellitus

4. Expert Committee on the Diagnosis and Classification of Diabetes 15. Kim G, Caprio S. Diabetes and insulin resistance in pediatric
Mellitus. Report of the expert committee on the diagnosis and obesity. Pediatr Clin North Am. 2011;58(6):1355–1361, ix
classification of diabetes mellitus. Diabetes Care. 1997;20(7): 16. Klingensmith GJ, Pyle L, Arslanian S, et al; TODAY Study
1183–1197 Group. The presence of GAD and IA-2 antibodies in youth with
5. Dabelea D, Bell RA, D’Agostino RB Jr, et al; Writing Group for a type 2 diabetes phenotype: results from the TODAY study.
the SEARCH for Diabetes in Youth Study Group. Incidence of Diabetes Care. 2010;33(9):1970–1975
diabetes in youth in the United States. JAMA. 2007;297(24): 17. Pinhas-Hamiel O, Zeitler P. Acute and chronic complications
2716–2724 of type 2 diabetes mellitus in children and adolescents. Lancet.
6. American Diabetes Association. Type 2 diabetes in children and 2007;369(9575):1823–1831
adolescents. Diabetes Care. 2000;23(3):381–389 18. American Diabetes Association. Standards of medical care in
7. Miller J, Silverstein JH, Rosenbloom AL. Type 2 diabetes in the diabetes—2012. Diabetes Care. 2012;35(suppl 1):S11–S63
child and adolescent. In: Lifshitz F, ed. Pediatric Endocrinology. 19. Rosenbloom AL, Silverstein JH, Amemiya S, Zeitler P,
Vol 1. Obesity, Diabetes Mellitus, Insulin Resistance, and Klingensmith GJ; International Society for Pediatric and Adolescent
Hypoglycemia. 5th ed. New York, NY: Informa Healthcare Inc; Diabetes. ISPAD Clinical Practice Consensus Guidelines 2006–
2007:168–187 2007: type 2 diabetes mellitus in the child and adolescent. Pediatr
8. Pettitt DJ, Aleck KA, Baird HR, Carraher MJ, Bennett PH, Diabetes. 2008;9(5):512–526
Knowler WC. Congenital susceptibility to NIDDM: role of in- 20. Copeland KC, Silverstein J, Moore KR, et al; American
trauterine environment. Diabetes. 1988;37(5):622–628 Academy of Pediatrics. Management of newly diagnosed type 2
9. Pettitt DJ, Forman MR, Hanson RL, Knowler WC, Bennett diabetes mellitus (T2DM) in children and adolescents. Pediatrics.
PH. Breastfeeding and incidence of non-insulin-dependent diabetes 2013;131(2):364–382
mellitus in Pima Indians. Lancet. 1997;350(9072):166–168 21. Zeitler P, Hirst K, Pyle L, et al; TODAY Study Group. A
10. American Diabetes Association. Diagnosis and classification of clinical trial to maintain glycemic control in youth with type 2
diabetes mellitus. Diabetes Care. 2012;35(1 Suppl 1):S64–S71 diabetes. N Engl J Med. 2012;366(24):2247–2256
11. Chen L, Appel LJ, Loria C, et al. Reduction in consumption of 22. Olansky L, Marchetti A, Lau H. Multicenter retrospective
sugar-sweetened beverages is associated with weight loss: the assessment of thiazolidinedione monotherapy and combination
PREMIER trial. Am J Clin Nutr. 2009;89(5):1299–1306 therapy in patients with type 2 diabetes: comparative subgroup
12. Welsh JA, Sharma AJ, Grellinger L, Vos MB. Consumption of analyses of glycemic control and blood lipid levels. Clin Ther. 2003;
added sugars is decreasing in the United States. Am J Clin Nutr. 25(suppl B):B64-B80.
2011;94(3):726–734 23. Genuth S, Alberti KG, Bennett P, et al; Expert Committee on
13. Phipps K, Barker DJ, Hales CN, Fall CH, Osmond C, Clark the Diagnosis and Classification of Diabetes Mellitus. Follow-up
PM. Fetal growth and impaired glucose tolerance in men and report on the diagnosis of diabetes mellitus. Diabetes Care. 2003;26
women. Diabetologia. 1993;36(3):225–228 (11):3160–3167
14. Vignini A, Raffaelli F, Cester A, et al. Environmental and 24. Adler AI. UKPDS-modelling of cardiovascular risk assessment
genetical aspects of the link between pregnancy, birth size, and type and lifetime simulation of outcomes. Diab Med. 2008;25(suppl 2):
2 diabetes. Curr Diabetes Rev. 2012;8(3):155–161 41–46

Parent Resources From the AAP at HealthyChildren.org


• English: http://www.healthychildren.org/English/health-issues/conditions/chronic/Pages/Type-2-Diabetes-A-
Manageable-Epidemic.aspx
• Spanish: http://www.healthychildren.org/spanish/health-issues/conditions/chronic/paginas/type-2-diabetes-a-
manageable-epidemic.aspx
• English: http://www.healthychildren.org/English/news/Pages/AAP-Publishes-First-Guidelines-to-Manage-Type-2-
Diabetes-in-Children.aspx

Pediatrics in Review Vol.34 No.12 December 2013 547


endocrinology diabetes mellitus

PIR Quiz
This quiz is available online at http://pedsinreview.org. NOTE: Learners can take Pediatrics in Review quizzes and claim credit online only. No paper
answer form will be printed in the journal.

New Minimum Performance Level Requirements


Per the 2010 revision of the American Medical Association (AMA) Physician’s Recognition Award (PRA) and credit system, a minimum performance
level must be established on enduring material and journal-based CME activities that are certified for AMA PRA Category 1 CreditTM. In order to
successfully complete 2013 Pediatrics in Review articles for AMA PRA Category 1 CreditTM, learners must demonstrate a minimum performance level
of 60% or higher on this assessment, which measures achievement of the educational purpose and/or objectives of this activity.
In Pediatrics in Review, AMA PRA Category 1 CreditTM may be claimed only if 60% or more of the questions are answered correctly. If you score less
than 60% on the assessment, you will be given additional opportunities to answer questions until an overall 60% or greater score is achieved.

1. Among the following factors, which is most likely to protect a 13-year-old patient against type 2 diabetes
mellitus (DM)?
A. Adolescent age.
B. Female sex.
C. History of being breastfed.
D. Mother who had gestational diabetes.
E. Native American race.

2. It can be difficult to distinguish between type 1 and type 2 DM in a child who presents with ketoacidosis.
Among the following, which clinical finding is most suggestive of type 2 DM?
A. Acanthosis nigricans.
B. Polydipsia.
C. Polyphagia.
D. Polyuria.
E. Weight loss.

3. In your management of a 7-year-old boy with type 1 DM, which of the following parameters are you likely to
assess every 3 months?
A. Albuminuria.
B. Hemoglobin A1c.
C. Lipid profile.
D. Liver function tests.
E. Signs of sleep apnea.

4. In addition to diet and exercise, you are considering pharmacotherapy for a 15-year-old girl who has had type
2 DM for the past year. Among the following, which is the only drug approved by the US Food and Drug
Administration for someone her age?
A. Acarbose.
B. Exenatide.
C. Glipizide.
D. Metformin.
E. Pioglitazone.

5. In a patient with newly diagnosed type 2 DM, initiation of insulin therapy is recommended if which one of the
following findings is present?
A. Fasting blood glucose level of 140 mg/dL (7.8 mmol/L).
B. Hemoglobin A1c level of 8.0% (0.08).
C. Hemoglobin A1c level of 8.5% (0.09).
D. Random blood glucose level of 200 mg/dL (11.1 mmol/L).
E. Random blood glucose 275 mg/dL (15.3 mmol/L).

548 Pediatrics in Review Vol.34 No.12 December 2013


Article nutrition

Managing Feeding Problems and Feeding


Disorders
James A. Phalen, MD*
Educational Gap
Up to 50% of typically developing children and up to 80% of those who have develop-
Author Disclosure mental disabilities have feeding problems. These may evolve into a feeding disorder, with
Dr Phalen has potential effects on psychomotor and neurologic development. (1) (2)
disclosed no financial
relationships relevant
Objectives After completing this article, readers should be able to:
to this article. This
commentary does not 1. Understand normal feeding patterns in children.
contain a discussion of 2. Recognize that feeding problems are common.
an unapproved/ 3. Prevent or ameliorate feeding problems.
investigative use of 4. Distinguish between feeding problems and feeding disorders.
a commercial product/ 5. Treat a child who has a feeding disorder.
device.
Introduction
Feeding plays a central role in the parent-infant relationship. The developmental progres-
sion of food selectivity is primarily determined by a child’s ability to manipulate, chew, and
swallow food (Table 1). Functional, safe feeding requires coordination of sensorimotor
function, swallowing, and breathing. Children self-regulate and may vary their oral intake
up to 30% per day with no ill effect on growth. Caregivers are responsible for what, when,
and where their children eat; the child is responsible for how much and whether they eat.
Normal feeding depends on the successful interaction of a child’s health, development,
temperament, experience, and environment. Altering any of these factors can result in
a feeding problem. (1)

Common Feeding Problems


Symptoms of feeding problems include food refusal, regurgitation, gagging, or swallowing
resistance (Table 2). (1) (3) Although the child maintains adequate growth, the behavior
causes distress for caretakers. Factors that increase a child’s risk for feeding problems, par-
ticularly during transition to more advanced textures, are listed in Table 3.
Between 25% and 50% of typically developing children and up to 80% of those with de-
velopmental disabilities have feeding problems. However, these problems are usually transient
and cause no serious outcomes. (1) Feeding problems are thus the norm. Practitioners must
consider cultural and ethnic differences and adjust for prematurity when setting expectations
for feeding. Some fundamental mealtime rules apply to toddlers and older children and can
prevent or resolve many feeding problems (Table 4). If a child is otherwise healthy and grow-
ing well, practitioners can reassure caregivers.

Abbreviations Feeding Disorder


BMI: body mass index A feeding disorder is any condition in which a child has an
CDC: Centers for Disease Control and Prevention inability or difficulty in eating or drinking sufficient quanti-
GERD: gastroesophageal reflux disease ties to maintain optimal nutritional status, regardless of
g-tube: gastrostomy tube cause. Growth may be unaffected. Between 3% and 10%
W/L: weight-length ratio of children are affected. Feeding disorders are multifactorial
and may begin with the child (Table 3), the parents, or the

*Developmental Pediatrics, San Antonio Military Pediatric Center, San Antonio, TX; Department of Pediatrics, University of Texas
Health Science Center at San Antonio; Department of Pediatrics, Uniformed Services University of the Health Sciences, Bethesda,
MD.

Pediatrics in Review Vol.34 No.12 December 2013 549


nutrition feeding problems and disorders

Table 1. Developmental Progression of Food Selectivity Based on Motor


Skills
Age, mo Food Consistency Fine and Gross Motor Skill Oral Motor Skill
0-4 Liquid Dependent on outside support Suckling present
Head control emerging Protective reflexes
4-6 Infant cereal Sits briefly Suckles more efficiently
Puréed fruits and vegetables Head control improves Sucks foods rather than phasic biting
Brings hands to midline Eats messily from spoon
Clasps bottle but needs help
6-9 Puréed meats Independent sitting Sips messily from cup
Variety of puréed baby food Reaches for food Vertical munching
Begins to finger feed Limited lateral tongue action
Unrefined pincer grasp Clears spoon with upper lip
Holds bottle independently Bite and release pattern
Assists with spoon Breaks off pieces of meltable solids
9-12 Ground and lumpy purées Sits in variety of positions Lip closure for liquids and soft solids
Mashed table foods Refined pincer grasp Spoon clearing more efficient
Soft, dissolvable solids Finger feeding refined Cup drinking with assistance
Grasps spoon with whole hand Begins drinking from
spouted cup
Grasps cup handle Begins to drink through a straw
12-18 Finely chopped table foods Scoops food Lateral tongue action
Chews juicy foods Brings to mouth Diagonal chewing
Bites through crunchy More independent feeding Begins drinking from straw
foods (cookies, crackers)
Upper teeth clear food from lower lip
18-24 More chewable solids Handles finger foods, spoon, Rotary chewing
and cup largely independently
Booster chair Cup drinking improved
Minimal food lost during eating
24-36 Tougher solids Total self-feeding Mature chewing for tougher solids
Open cup drinking without spilling
Variety of liquids through straw
Tongue clears food from lips
‡36 Advanced textures Begins using fork to stab food Open cup independently
(meats, fried foods,
whole fruits)

environment (Table 5). (2) Regardless of its origins, it BMI below the fifth percentile. Failure to thrive usually
affects the parent-child dyad and often evolves into a be- results from inadequate energy intake but may reflect inad-
havioral problem. Older children may experience low equate nutrient absorption or increased energy require-
self-esteem and social isolation. ments. Although growth charts exist for specific conditions,
the Centers for Disease Control and Prevention (CDC)
Evaluation of Feeding Disorders recommend that practitioners use the World Health Orga-
Initial evaluation of feeding disorders begins with the pri- nization growth standards to monitor growth for infants
mary care practitioner, who should assess parental coping, and children ages 0 to 2 years in the United States (avail-
mental health, and bonding. Most parents whose children able at http://www.cdc.gov/GrowthCharts/who_charts.
have feeding disorders describe feeling frustrated and dis- htm) and CDC growth charts to monitor growth for chil-
tressed at mealtime. The growth chart (length, weight, dren ages 2 to 20 years in the United States (available at
and weight-length ratio [W/L], or body mass index http://www.cdc.gov/GrowthCharts/cdc_charts.htm).
[BMI]) should be reviewed. True failure to thrive is a sus- A thorough review of the child’s prenatal, birth, and
tained decrease in growth velocity, best defined as a W/L or medical histories should focus on the following key areas:

550 Pediatrics in Review Vol.34 No.12 December 2013


nutrition feeding problems and disorders

• Aspiration. Aspiration involves passage of secretions,


Common Feeding
Table 2. drink, or solid food below the true vocal cords. It
may occur before, during, or after swallowing or from
Problems in Children gastroesophageal reflux. Signs include coughing,
Delayed development of oral motor and self-feeding throat clearing, gurgling voice, noisy breathing, recur-
skills; common in infants and children with hypotonia, rent wheezing or stridor, and recurrent lower respira-
global developmental delay or intellectual disability, tory tract infections. Some children aspirate with no
and neurologic disorders obvious symptoms; this condition is called silent aspi-
Reluctance or refusal to eat based on sensory issues (eg,
ration and suggests lack of a protective reflex. (7) As-
taste, texture, temperature, smell, or appearance)
Food selectivity (eg, personal preference, discomfort with piration suggests oral motor delay or oropharyngeal
certain foods because of gastroesophageal reflux dysphagia. Infants and children who have oral motor
disease, or food allergy) delay typically have oral hypotonia and an underdevel-
Decreased appetite for or interest in food oped suck-swallow-breathe pattern. Thus, they may
Slow feeding (ie, >30 minutes to finish)
Food pocketing (ie, holding food in cheeks or front of
have poor lip closure, drooling after age 12 months,
mouth for prolonged periods) suggests poor oral lack of tongue lateralization, and loss of food from
transport or refusal the mouth (Table 1). Oropharyngeal dysphagia, how-
Using feeding behaviors to comfort, self-soothe, or ever, is pathologic difficulty swallowing because of un-
self-stimulate derlying neurologic or structural abnormalities.
• Motor disabilities (eg, cerebral palsy and spina bifida). Chil-
dren with motor disabilities are less mobile than neurotyp-
• Small for gestational age. Up to 15% of infants born ical children and thus have lower energy requirements. A
small for gestational age fail to achieve appropriate W/L or BMI greater than the 50th percentile makes hy-
catch-up growth by age 2 years and continue to expe- giene, mobility, and transfers (eg, wheelchair to tub) more
rience poor growth throughout childhood. Rapid challenging and increases the risk of medical complications
catch-up growth before age 2 years in this group in- of obesity through excessive caloric intake.
creases the risk of developing metabolic disease later • Gastroesophageal reflux disease (GERD). Signs of
in life. (4) (5) (6) Thus, practitioners must temper at- GERD include regurgitation, postprandial emesis,
tempts to promote catch-up growth against the risks. choking, gagging, food refusal, constant or sudden
It is reasonable to aim for a W/L or BMI between the crying, irritability, poor sleep patterns, apnea, stridor,
10th and 50th percentiles in this population. laryngospasm, bronchospasm, and hoarseness. Eosino-
philic esophagitis deserves consideration in any child
presenting with symptoms of GERD in whom a trial
Pediatric Conditions
Table 3. of medical therapy with a proton pump inhibitor fails,
Associated With Feeding especially in the setting of atopy. Persistent symptoms
and food impaction (food getting stuck in the esoph-
Problems and Feeding Disorders agus) should raise additional concern.
Temperamental traits that complicate feeding and • Constipation. Signs and symptoms of constipation in-
overwhelm parents clude bulky, painful, or infrequent bowel movements,
Prematurity (especially neonates who require prolonged failed attempts to stool, bloody stools, anal fissures,
respiratory support or enteral feeds or with delayed urinary incontinence, and overflow incontinence
introduction of oral feeds) (encopresis). Parents often confuse the latter with diar-
Genetic or chromosomal abnormalities (eg, Down
syndrome and inherited neuromuscular disease) rhea. Chronic constipation may cause early satiety and
Craniofacial anomalies (eg, Pierre-Robin sequence and reduced caloric intake. Stool withholding exacerbates
cleft palate) constipation and may have psychosocial consequences.
Acquired brain impairment (eg, cerebral palsy, stroke, • Medications. Medications that can cause excessive se-
and traumatic brain injury)
dation or decreased appetite include stimulants, selec-
Gastrointestinal disorders (eg, gastroesophageal reflux
disease and chronic constipation) tive serotonin reuptake inhibitors, and topiramate.
Neurodevelopmental disorders (eg, autism spectrum
Pica and rumination are more likely to occur in individuals
disorder, global developmental delay, and intellectual
disability) who have developmental disabilities, psychiatric disorders, or
physiologic conditions (eg, iron deficiency and pregnancy).

Pediatrics in Review Vol.34 No.12 December 2013 551


nutrition feeding problems and disorders

Table 4. Mealtime Rules for Toddlers and Older Children


Feature Rules Benefit
Scheduling Regular meals with planned, low-calorie snacks Prevents “grazing”
Same room, table, and utensils for every meal Enhances sense of hunger and satiety
Limit mealtime to 30 minutes Caregivers maintain control of
(ie, “kitchen is open”) feeding schedule
Offer no liquids between meals except plain Home is less chaotic
water (ie, “kitchen is closed”)
Environment Family sits together at mealtime Focus is on socializing rather than eating
Neutral atmosphere (no forced feeding or Avoids conflict
comments regarding intake)
Eliminate distractions: turn off all Allows child to focus on mealtime
electronic devices, child sits with
back to open room
Allow younger child to explore foods by Mealtime is more pleasant
touching, smelling, and tasting
Allow older child to participate in food
purchase and meal preparation
Never use food as a reward, bribe, or incentive
Praise child for showing interest in food
Allow at least 20 exposures to new foods for
acceptance
Methods Optimal feeding posture: More likely to consume calorically dense foods
• Head midline and neck neutral or Expands food repertoire
slightly flexed
• Trunk symmetrical and elongated Promotes independence
• Pelvis stable with hips symmetrical in neutral Prevents constipation and anemia from
position excessive milk intake
• Hips, knees, and ankles each at 90o Prevents loose stools and dental caries from
excessive juice intake
Serve food at table for everyone from same
container
Small portions
Small easily chewed bites or long thin
strips child can grasp
Offer liquids only after child begins eating solids
Offer plain, unflavored water as primary beverage
Limit daily intake of low-fat or fat-free white or
flavored milk to:
• 2 cups for children ages 2 to 3 years
• 2½ cups for children ages 4 to 8 years
• 3 cups for those 9 years and older
Do not dilute fruit juice, and limit to
4 to 6 oz per day
Discourage sweetened beverages (soft
drinks and sports or energy drinks)
Encourage self-feeding (eg, finger feeding
and holding spoon)
Food chaining: offer unfamiliar or
nonpreferred foods first and paired with
familiar or preferred foods
Avoid excessive coaxing, threatening, or
forced feeding
Remove food without comment if child
loses interest
Wipe face and clean up only when
meal completed

552 Pediatrics in Review Vol.34 No.12 December 2013


nutrition feeding problems and disorders

Parental and Environmental Factors Associated With Feeding


Table 5.

Disorder
Factor Result
Conditioned aversion Pairing eating with a painful medical condition or procedure (eg, airway suctioning and
intubation)
Lack of opportunity Delayed introduction of breast, bottle, or solids is associated with delayed attainment of
appropriate eating skills
Positive reinforcement Caretakers coax or bribe infant who bats away the spoon, turns the head away, or cries
Negative reinforcement Caretakers terminate meal when child acts out
Forced feeding Results in aversion to meals and evokes inappropriate behavior at future meals
Overly rigid parents Undermines child’s ability to regulate food intake and impairs child’s psychosocial development
Chaotic parents Fail to provide child with appropriate food, support, structure, or opportunity to learn to enjoy
a variety of foods or to master eating-related social patterns

Neither should be diagnosed unless symptoms are of an and neurologic development. It may also affect the im-
unusual extent or cause health concerns. (8) (9) mune, skeletal, and cardiovascular systems.
Practitioners should select diagnostic laboratory stud-
• Pica disorder. Pica disorder is the recurring ingestion of
ies based on the history and physical examination find-
nonfood, nonnutritive substances for at least 1 month in
ings. The following are reasonable:
a child at least 2 years of age, which is inappropriate to
the child’s developmental level and sociocultural norms. • In cases of failure to thrive: complete blood cell count,
• Rumination disorder. Rumination disorder is the re- urinalysis, blood urea nitrogen, serum electrolytes, and
peated regurgitation of food for at least 1 month. Re- serologic screening for celiac disease (usually IgA anti-
gurgitated food may be rechewed, reswallowed, or spit bodies to tissue transglutaminase).
out, most often during or shortly after meals. It is not • In cases of pica disorder: serum iron and lead levels.
associated with nausea or a medical condition. It is vo-
litional, distinguishing it from vomiting and gastro-
Classification of Feeding Disorders
esophageal reflux.
The Diagnostic and Statistical Manual of Mental Dis-
orders, Fifth Edition (DSM-5), informed by available
Feeding History research and extensive discussion of expert clinical expe-
It is important to have caretakers describe the mealtime en- rience and opinion, takes a lifespan approach to how age
vironment (Table 5) and the child’s feeding habits (Table 6). and development affect psychiatric diagnoses. Avoidant/
restrictive food intake disorder replaces the previous term
Feeding Observation feeding disorder of infancy or early childhood (Table 7).
If time and resources allow, the practitioner or clinic Other classification systems exist; however, none is uni-
nurse may observe (in person or by video) the child feed- versally accepted, and few are evidence based.
ing. Such observation allows identification of appropriate
child positioning and posture, the child’s hunger and sa-
tiety cues, the caretaker’s response to and interactions
Management of Feeding Disorders
The long-term goals of treatment are to improve nutritional
with the child, any delayed oral motor or self-feeding
status, growth, feeding safety, and quality of life. Recognition
skills, and difficulty managing or tolerating liquids or sol-
and treatment of GERD and constipation are essential. On
ids (eg, oropharyngeal dysphagia).
the basis of findings, practitioners may consult the following:
A complete physical, neurologic, and oral motor ex-
amination must be performed. The oral motor examina- • A pediatric speech-language pathologist to perform
tion includes evaluating facial symmetry, hard and soft a clinical swallowing evaluation coupled with a video
palate for (submucous) cleft, and dentition; symmetry fluoroscopic swallow study to evaluate for oral motor
and movement of lips and tongue; vocal intensity, pitch, delay and oropharyngeal dysphagia.
and quality; and cranial nerves. Prolonged inadequate en- • A registered pediatric dietitian to assess caloric intake,
ergy and nutrient intake may have broad effects beyond nutritional quality, and dietary practices and to coman-
physical growth, with potential effects on psychomotor age enteral feeds.

Pediatrics in Review Vol.34 No.12 December 2013 553


nutrition feeding problems and disorders

Table 6. Components of a Feeding History


Ask Caretakers Clinical Significance
How they prepare infant formula Healthy infants require a concentration of 20 kcal/oz,
whereas those who have medical problems (eg,
cardiac disease) or failure to thrive may require
more concentrated or specialized formula
Whether they add infant cereal, puréed solids, or Poor tolerance of nonthickened formula may indicate
proprietary thickeners to formula oral motor delay or oropharyngeal dysphagia;
premature introduction of solids may reflect
cultural practices
About food preferences and nutritional deficits (eg, convenience May suggest the child’s preferences or that caretakers
foods, inadequate intake of fruits and vegetables, and excessive have difficulty setting limits
juice or milk intake)
About grazing (eg, overly frequent breastfeeding in older infants; Grazing may lead to reduced energy intake, increases
toddlers and older children eating and drinking throughout the the risk for dental caries, and suggests caretakers
day); these children may come to your clinic snacking and have difficulty setting limits
drinking.
About reliance on dietary supplements (eg, multivitamins, Indicates caretaker or practitioner concern and may
megavitamins, toddler formula, and breakfast drinks) or appetite reveal inappropriate feeding practices
stimulants (eg, megestrol acetate and cyproheptadine)
About difficulty chewing, excessive drooling, or food Indicates delayed oral motor skills
or liquid leaving the mouth or nose
Patient’s age at and difficulty with transitions May indicate delayed oral motor skills or behavioral
from liquids to purées to solids preferences
Whether child gags, chokes, coughs, or vomits during feeds or has Raises concern for oropharyngeal dysphagia
disruptions in breathing, apnea, or cyanosis during feeds
About refusal, tantrums, rumination, pica, avoidance of certain Identifies maladaptive mealtime behaviors
food textures, temperatures, and colors

• A pediatric gastroenterologist to evaluate severe recal- Dietary interventions aim to establish a balanced,
citrant constipation, GERD, and eosinophilic esopha- healthful diet. Because liquids are usually easier than sol-
gitis and to comanage enteral feeds. ids to consume, the tendency is to supplement the diet
• A developmental pediatrician to further evaluate for with toddler formula. Often formulas come to replace
contributing causes (eg, global developmental delay, meals, leading to grazing and inadequate energy and nu-
autism spectrum disorder, and parent-child conflict). trient intake, further aggravating the child’s nutritional
• An interdisciplinary feeding team that includes some deficiency. Clinicians should thus discourage overreliance
combination of the above professionals along with on toddler formulas and other liquid supplements. Reg-
a clinical child and pediatric psychologist. istered dietitians may recommend nutrient- and energy-
Oral motor skills usually improve over time but can be dense foods and/or specialized formula.
promoted in a more organized and efficient manner with Behavioral feeding therapy is implemented most ap-
therapy. Pediatric speech-language pathologists and oc- propriately in the context of an interdisciplinary team,
cupational therapists generally use noninvasive treat- typically including a registered dietitian, speech-language
ments, such as proper positioning and posture, thickened pathologist, and clinical child and pediatric psychologist.
liquids, modification of bolus size, oral motor and desen- Effective therapy aims to eliminate factors that reinforce
sitization exercises, specialized nipples and bottles, and maladaptive mealtime behavior. (2) Settings include out-
altering the temperature, texture, or presentation of food. patient, partial day, and inpatient facilities. Treatment
The evidence base for these interventions is limited. (7) should start with the least intrusive approach, generally
(10) Transcutaneous neuromuscular electrical stimula- outpatient. The literature does not support pharmaco-
tion is an emerging therapy for dysphagia in children. logic treatment with appetite stimulants (eg, megestrol
It involves noninvasive, external electrical stimulation acetate and cyproheptadine) for behavioral feeding disor-
of peripheral motor nerves of the anterior throat to acti- ders. Caregiver compliance is strongly associated with
vate the pharyngeal muscles involved in swallowing. skills maintenance and generalization.

554 Pediatrics in Review Vol.34 No.12 December 2013


nutrition feeding problems and disorders

enteral nutrition is emotionally challenging for parents, it


DSM-5 Diagnostic Criteria
Table 7. eliminates the pressure for oral feeding. It allows the child
to be fed safely and efficiently, reducing the risk of aspi-
for Avoidant/Restrictive Food ration and allowing for catch-up growth. Rapid or volu-
Intake Disorder (307.59) minous feeds may trigger retching or aggravate GERD.
Excessive caloric intake can cause overweight or obe-
A. An eating or feeding disturbance (eg, apparent lack of sity, leading to problems handling and lifting children
interest in eating or food; avoidance based on the who have physical disabilities. Bypassing the oral route
sensory characteristics of food; concern about
aversive consequences of eating) as manifested by deprives the child of the experiences associated with feed-
persistent failure to meet appropriate nutritional and/ ing, thus delaying oral sensorimotor skills and increasing
or energy needs associated with one (or more) of the the risk for sensory-based food aversions when oral feeds
following: are reintroduced. The earlier in life that a g-tube is placed,
1. Significant weight loss (or failure to achieve the more difficult it becomes to wean the child from it
expected weight gain or faltering growth in children)
2. Significant nutritional deficiency later in life. Finally, continuous feeds are less physiologic
3. Dependence on enteral feeding or oral nutritional than are bolus feeds, resulting in decreased appetite and
supplements increasing the risk of grazing and reliance on the g-tube.
4. Marked interference with psychosocial functioning Tube dependency occurs when the child has the ability to
B. The disturbance is not better explained by lack of ingest and digest food but cannot be weaned from tube
available food or by an associated culturally
sanctioned practice. feeding, regardless of medical criteria. For these reasons,
C. The eating disturbance does not occur exclusively children who have g-tubes should be exposed to the
during the course of anorexia nervosa or bulimia mealtime environment, be encouraged to touch and in-
nervosa, and there is no evidence of a disturbance in teract with food without regard to intake, be given bolus
which one’s body weight or shape is experienced.
feeds if tolerated, and have oral feeds advanced when pos-
D. The eating disturbance is attributable to a concurrent
medical condition or better explained by another sible. This, along with oral motor and/or behavioral
mental disorder. When the eating disturbance occurs feeding therapy involving the parents, helps the child
in the context of another condition or disorder, the progress to g-tube independence. Children who receive
severity of the eating disturbance exceeds that no feeds, fluids, or flushes through their g-tube for 12
routinely associated with the condition or disorder months are candidates to have the device removed. Pre-
and warrants additional clinical attention.
mature removal may increase the child’s risk for compli-
Reprinted with permission from the American Psychiatric Association. cations, such as failure to thrive.
Diagnostic and Statistical Manual of Mental Disorders, Fifth Edition.
Washington, DC: American Psychiatric Association; 2013.

Summary
Those who cannot consume sufficient energy and nu-
trients or do so safely by mouth require enteral (ie, tube) • On the basis of strong research evidence, feeding
nutrition. Enteral nutrition can be delivered via nasogas- problems and feeding disorders are common,
tric tube, orogastric tube, or gastrostomy tube (g-tube). especially in children who have developmental
disabilities. (1) (3)
For those requiring enteral nutrition for longer than 6 • On the basis of strong research evidence, a variety of
weeks, the latter is preferred. Minimally invasive percuta- prenatal, medical, environmental, behavioral, and
neous endoscopic gastrostomy and laparoscopic gastro- parental factors contribute to childhood feeding
stomy have largely supplanted the open laparotomy for disorders. (1) (3)
placement of g-tubes. To preserve oral activity and feed- • On the basis of some research evidence plus
consensus, many feeding problems are preventable or
ing habits, along with hunger and satiety cues, oral feeds easily treated.
(when safe) should precede supplemental tube feeds. En- • On the basis of strong research evidence, left
teral nutrition is delivered either intermittently or contin- untreated, feeding disorders may result in
uously. The preferred method is intermittent bolus complications, including aspiration pneumonitis,
feedings, which is more physiologic; however, if the pa- failure to thrive, and parent-child conflict.
• On the basis of some research evidence plus consensus,
tient does not tolerate bolus feeds then continuous feeds, treatment of feeding disorders improves nutritional
either intragastric or transpyloric (through a gastrojeju- status, growth, feeding safety, and quality of life.
nostomy), is reasonable. Although the decision to initiate

Pediatrics in Review Vol.34 No.12 December 2013 555


nutrition feeding problems and disorders

ACKNOWLEDGMENT. The author acknowledges past 3. Eicher PS. Feeding and its disorders. In: Batshaw M, ed.
and present members of the Interdisciplinary Feeding Children With Disabilities. 7th ed. Baltimore, MD: Brookes
Publishing; 2012
Team at San Antonio Military Pediatric Center for their
4. Ong KK, Loos RJ. Rapid infancy weight gain and subsequent
clinical expertise and guidance, without which this article obesity: systematic reviews and hopeful suggestions. Acta Paediatr.
would not be possible. 2006;95(8):904–908
(The views expressed are those of the author and do not 5. Baird J, Fisher D, Lucas P, Kleijnen J, Roberts H, Law C. Being
reflect the official policy or position of the US Air Force, De- big or growing fast: systematic review of size and growth in infancy
and later obesity. BMJ. 2005;331(7522):929
partment of Defense or the US Government.)
6. Monteiro POA, Victora CG. Rapid growth in infancy and child-
Suggested Reading hood and obesity in later life—a systematic review. Obes Rev. 2005;6
American Speech-Language-Hearing Association. Swallowing and (2):143–154
Feeding Disorders. Available at http://www.asha.org/slp/ 7. Miller CK. Aspiration and swallowing dysfunction in pediatric
clinical/dysphagia/. Accessed October 29, 2013 patients. Infant Child Adolesc Nutr. 2011;3:336–343
Ellyn Satter Associates. Available at http://www.EllynSatter.com. 8. Uher R, Rutter M. Classification of feeding and eating disorders:
Accessed October 29, 2013 review of evidence and proposals for ICD-11. World Psychiatry.
2012;11(2):80–92
References 9. American Psychiatric Association. Diagnostic and Statistical
1. Bryant-Waugh R, Markham L, Kreipe RE, Walsh BT. Feeding and Manual of Mental Disorders, Fifth Edition. Washington, DC:
eating disorders in childhood. Int J Eat Disord. 2010;43(2):98–111 American Psychiatric Association; 2013
2. Sharp WG, Jaquess DL, Morton JF, Herzinger CV. Pediatric 10. Morgan AT, Dodrill P, Ward EC. Interventions for oropha-
feeding disorders: a quantitative synthesis of treatment outcomes. ryngeal dysphagia in children with neurological impairment.
Clin Child Fam Psychol Rev. 2010;13(4):348–365 Cochrane Database Syst Rev. 2012;10:CD009456

PIR Quiz
This quiz is available online at http://pedsinreview.org. NOTE: Learners can take Pediatrics in Review quizzes and claim credit online only. No paper
answer form will be printed in the journal.

New Minimum Performance Level Requirements


Per the 2010 revision of the American Medical Association (AMA) Physician’s Recognition Award (PRA) and credit system, a minimum performance
level must be established on enduring material and journal-based CME activities that are certified for AMA PRA Category 1 CreditTM. To successfully
complete 2013 Pediatrics in Review articles for AMA PRA Category 1 CreditTM, learners must demonstrate a minimum performance level of 60% or
higher on this assessment, which measures achievement of the educational purpose and/or objectives of this activity.
In Pediatrics in Review, AMA PRA Category 1 CreditTM may be claimed only if 60% or more of the questions are answered correctly. If you score less
than 60% on the assessment, you will be given additional opportunities to answer questions until an overall 60% or greater score is achieved.

1. A previously well 7-month-old infant is developing normally. She sits and holds her bottle independently,
reaches for food, and finger feeds herself. What is the most appropriate food choice for this infant?
A. Chewable solids.
B. Finely chopped table foods.
C. Mashed table foods.
D. Pureed meats.
E. Whole fruits.

The vignette below will be used for questions 2 and 3.


2. A healthy 14-month-old boy refuses solids at mealtime and prefers milk and juice. He has grown and
developed normally. He has no siblings. His mother is frustrated and concerned because nothing has worked to
change the behavior. His physical examination findings are unremarkable. What is the first step in treatment?
A. Ordering a comprehensive metabolic panel.
B. Prescribing a proton pump inhibitor.
C. Reassuring the mother.
D. Referring to a gastroenterologist.
E. Referring to an occupational therapist.

556 Pediatrics in Review Vol.34 No.12 December 2013


nutrition feeding problems and disorders

3. The mother of the 14-month-old boy asks for some healthful behavior-shaping tips. In response you would
have her:
A. Arrange special mealtimes for her son.
B. Encourage him to self-feed.
C. Make the child sit alone at the table until his plate is clean.
D. Offer a cup of diluted juice if he eats some solids.
E. Permit the boy to watch videos during mealtime.

4. Which of the following 14-month-old boys who are growing normally has a feeding disorder rather than just
a feeding problem?
A. Drools constantly and dribbles food from mouth.
B. Feeds slowly but finishes most meals.
C. Pockets foods in mouth he does not like.
D. Refuses solids but loves milk and juice.
E. Spits out lima beans and broccoli.

5. A 14-month-old girl has severe oropharyngeal dysphagia related to hypoxic-ischemic brain injury secondary to
abruptio placentae. She now requires enteral feeding to provide adequate nutrition. Assuming the gut works
normally, the child is exposed to the family mealtime environment, and the child is encouraged to touch food
without regard to intake, optimal management would include:
A. Continuous gastrostomy tube feeding.
B. Continuous gastrojejunostomy tube feeding.
C. Continuous nasogastric tube feeding.
D. Intermittent bolus gastrostomy tube feeding.
E. Intermittent bolus nasogastric tube feeding.

Parent Resources From the AAP at HealthyChildren.org


• English: http://www.healthychildren.org/English/ages-stages/baby/feeding-nutrition/Pages/Signs-of-Feeding-
Difficulties.aspx
• Spanish: http://www.healthychildren.org/spanish/ages-stages/baby/feeding-nutrition/paginas/signs-of-feeding-
difficulties.aspx

Pediatrics in Review Vol.34 No.12 December 2013 557


Article infectious diseases

Human Metapneumovirus
Jennifer E. Schuster, MD,*
Educational Gaps
John V. Williams, MD*
1. Respiratory viruses are a leading cause of global pediatric morbidity and mortality.
Human metapneumovirus is the second leading cause of pediatric viral lower
Author Disclosure respiratory tract infection. Because human metapneumovirus is a newly recognized
Dr Schuster has pathogen, many clinicians are unfamiliar with the epidemiology, pathogenesis, and
disclosed no financial clinical signs and symptoms of infection.
relationships relevant 2. Knowledge of the biology of human metapneumovirus is important to understand the
to this article. Dr pathogenesis of human metapneumovirus and how best to treat it.
Williams has disclosed 3. Diagnosis of human metapneumovirus is challenging, and clinicians should be aware
that he serves on the of the sensitivity and specificity of available tests.
Scientific Advisory
Board of Quidel. This
Objectives After completing this article, readers should be able to:
commentary does
contain a discussion of 1. Discuss the current epidemiology of human metapneumovirus.
an unapproved/ 2. Recognize clinical manifestations of human metapneumovirus.
investigative use of 3. Identify populations most susceptible to human metapneumovirus.
a commercial product/ 4. Discuss laboratory studies available for the diagnosis of human metapneumovirus.
device.
Report of a Case
A 3-year-old girl undergoing induction chemotherapy for pre–B-cell acute lymphoblastic
leukemia presents with a history of temperatures to 38.9°C. She has a 2-day history of
cough and nasal congestion. She is refusing solid food and taking minimal liquids. She
had one episode of nonbloody, nonbilious emesis. On physical examination, she appears
ill and is having difficulty breathing. Her temperature is 39.5°C, heart rate is 130 beats
per minute, respiratory rate is 40 breaths per minute, blood pressure is 90/60 mm Hg,
and oxygen saturation on room air is 92%. She has a bulging, erythematous, nonmobile
right tympanic membrane and clear rhinorrhea. Her lung examination findings are notable
for tachypnea with retractions and diffuse wheezing.
Viral infection is suspected, but because she is undergoing chemotherapy, she merits
evaluation for serious bacterial infection. White blood cell count is 1200/mL (12.0 
109/L) (80% lymphocytes, 10% neutrophils, 10% monocytes), hemoglobin level is 8.9
mg/dL (89 g/L), and platelet count is 169  103/µL (169  109/L). Blood culture
is performed. Chest radiography reveals diffuse perihilar infiltrates. Because she is febrile
and neutropenic, broad-spectrum antibiotics are administered, and she is admitted to the hos-
pital. She receives intravenous fluids and oxygen via nasal cannula.
During the next 48 hours, her respiratory status worsens, and she requires intubation
and mechanical ventilatory support. Blood culture result re-
mains negative. After 5 days, her respiratory status improves,
Abbreviations and she is extubated. A nasopharyngeal wash result is posi-
tive for human metapneumovirus by polymerase chain reac-
HIV: human immunodeficiency virus
tion (PCR) testing and negative for other pathogens.
MPV: metapneumovirus
PCR: polymerase chain reaction
PIV: parainfluenza virus Introduction
RSV: respiratory syncytial virus Human metapneumovirus (MPV) is a respiratory pathogen
with worldwide prevalence that produces disease clinically

*Vanderbilt University School of Medicine, Nashville, Tennessee.

558 Pediatrics in Review Vol.34 No.12 December 2013


infectious diseases metapneumovirus

similar to respiratory syncytial virus (RSV). Although the seasons. Incidence varies from 5% to 20% and is generally
virus was not identified until 2001, antibodies to MPV lower than RSV. Rates of MPV are comparable to other
were detected in archived human sera, demonstrating that respiratory viruses, such as influenza and parainfluenza vi-
the virus has been circulating since at least the 1950s. Mul- rus (PIV) types 1 to 3 combined. One large, multicenter,
tiple reasons likely contributed to the delayed identifica- prospective study enrolled children with acute respiratory
tion of the virus. Trypsin supplementation is required infection among inpatient, emergency department, and
for growth in culture, and slow replication kinetics lead clinic settings; MPV was the second most common virus
to delayed cytopathic effect and identification in cell cul- after RSV in this study. In retrospective, multiyear, epidemi-
ture. Furthermore, many laboratory cell lines are not per- ologic studies, researchers have noted that one subgroup
missive for MPV infection. Within the last 12 years, may dominate, but this varies among geographic locations
researchers have defined the epidemiology of MPV, devel- and from year to year. Coinfection with other respiratory
oped rapid diagnostic testing, and are investigating host pathogens, such as rhinoviruses, RSV, PIV, and adenovirus,
immune responses to guide vaccine development. has been documented in a few MPV infections. Most stud-
ies have found that viral coinfections are not more severe
Virology clinically than MPV-alone infection. In addition, data from
MPV, like other members of the Paramyxovirus family, is animal and small human studies suggest that MPV may be
an enveloped, single-stranded, negative-sense RNA virus. associated with increased development of bacterial coinfec-
It is most closely related to avian metapneumovirus type C, tions with Streptococcus pneumoniae.
the other member of the Metapneumovirus genus, and it Large-scale evaluation of adult serum samples has dem-
is in the Pneumovirinae subfamily with RSV. The fusion onstrated that nearly all adults are seropositive for MPV. In
protein is required for attachment and entry and requires most geographic locations, seroprevalence is 100% in chil-
trypsin for cleavage to the active form. The other external dren older than 5 years. The mean age of children hospi-
proteins, glycoprotein and small hydrophobic protein, are talized with MPV-associated lower respiratory tract
not required for entry. The virus contains 9 structural pro- infection is 6 to 12 months, older than those hospitalized
teins. Integrins and heparan sulfate have been identified as with RSV. Maternal antibodies may provide protection to
host receptors. The genome is approximately 13 kb in young infants. Studies have demonstrated symptomatic
length. Phylogenetic analysis identifies 2 groups (A and subsequent infection with viruses from different subgroups
B), each with 2 subgroups (A1, A2, B1, and B2). Clinical in young children, although in animal models evidence
disease is similar for all subgroups. of serologic cross-protection occurs. MPV is less fre-
quently identified in older children, likely because of
this cross-protective immunity.
Viral Replication Prospective studies demonstrate that MPV is a leading
Similar to other respiratory viruses, MPV spreads by re- cause of viral respiratory infections in older adults, with hos-
spiratory droplets. The incubation period is thought to pitalization rates similar to influenza and RSV. The virus has
be 4 to 9 days, although in nonhuman primate models been identified as a cause of community-acquired pneumo-
a shorter period has been observed. Shedding occurs nia and chronic obstructive pulmonary disease exacerbations
for 7 to 14 days. Virus can remain infectious on fomites in adults. However, healthy young adults may demonstrate
for 8 hours, although viral RNA has been isolated from MPV seroconversion without clinical illness.
noninfectious particles up to 7 days after inoculation.
MPV has been implicated in both hospital and institutional Pathology
nosocomial outbreaks, emphasizing the importance of ap- Cotton rat and mouse models exhibit peribronchial in-
propriate precautions, particularly around immunocom- flammation with an increase in mononuclear and lym-
promised children. phocytic cells. Perivascular edema is present. The most
severe condition in these models has been noted on days
Epidemiology 5 to 7, although abnormalities are present on days 1 to
MPV has a worldwide prevalence, with the incidence 14. Nonhuman primate models demonstrate inflamma-
varying yearly and by geographic location. The virus tory and erosive changes in the airway mucosa. Replication
has been isolated year-round, but the peak seasonal inci- occurs only in ciliated epithelial cells of the respiratory
dence in temperate regions is February to April, later than tract; there is no evidence of viremia in humans or animal
the usual peak of RSV infection. In subtropical climates, models. Lung disease in humans is more difficult to char-
MPV is most prevalent during the spring and summer acterize because of barotrauma related to ventilation,

Pediatrics in Review Vol.34 No.12 December 2013 559


infectious diseases metapneumovirus

although increased alveolar macrophages are present, similar upper respiratory tract disease. Fever is present in up to
to nonhuman primates. 50% of children with MPV-associated respiratory tract in-
fection. Conjunctivitis has been identified in a small subset
Host Response of patients. Large epidemiologic studies have identified
Multiple studies have evaluated the cytokine response to MPV as a cause of upper respiratory tract infections at
MPV. Although a vigorous cytokine response is described, a similar rate as RSV, influenza, and PIV.
the interaction between the virus and production of differ- Acute otitis media is a common complication of MPV.
ent cytokines remains unclear. Despite the variable findings In one case series, almost one-quarter of children with
regarding the increase and decrease of assorted cytokines, MPV had acute otitis media. Another report noted that
several studies have demonstrated decreased interferon g 13% of children with acute otitis media had MPV-positive
secretion after infection. nasopharyngeal specimens. The virus has been isolated
Antibodies alone can protect against infection, includ- alone in tympanic fluid; however, in most cases, middle
ing cross-protection from infection with other subgroups. ear fluid cultures yield other bacterial pathogens, partic-
However, immunity appears to wane over time. In nonhu- ularly S pneumoniae.
man primates, low-level replication with secondary infec- Manifestations of lower respiratory tract infection in-
tion by a virus from the same subgroup occurred 12 clude cough, wheeze, rhonchi, and dyspnea. Epidemiologic
weeks after primary infection despite detectable serum studies have demonstrated that a greater number of chil-
antibodies. In the same study, no evidence of protection dren infected with MPV are diagnosed as having pneumo-
was seen when animals were challenged 11 months after nia vs bronchiolitis, especially when compared with RSV
primary infection. Subsequent infection in children has (Table 1). Hypoxia and cyanosis can occur with severe
been documented, usually with a virus from a different lower respiratory tract disease. Radiographic abnormalities
subgroup; most of these presented with lower respiratory include diffuse findings, such as perihilar infiltrates and alve-
tract illness during primary infection and upper respiratory olar disease, and focal findings, including bronchopneu-
tract illness during secondary MPV infection. Some studies monic changes, lobar pneumonia, and effusions (Fig 1).
have identified a decrease in MPV-specific antibody in el- MPV has been isolated in both children and adults
derly adults compared with younger adults. Additional hospitalized for asthma exacerbations. In one study, 7%
prospective studies have noted that baseline MPV anti- of children with asthma exacerbations had MPV isolated
body levels are lower in patients who subsequently become from nasopharyngeal swabs. Another large-scale epidemi-
infected with the virus compared with those who do not ologic study noted that 14% of children with MPV had
acquire infection. asthma as a discharge diagnosis. This finding was similar
Studies that evaluated cytotoxic T-cell responses have to the percentage of children with RSV and a discharge
identified responses to multiple proteins, including the diagnosis of asthma; however, rhinovirus was more fre-
fusion protein, which is highly conserved in subgroups. quently associated with asthma exacerbations in the same
However, recent studies suggest that MPV and other respi- study. Another study noted that MPV bronchiolitis in in-
ratory viruses cause impairment of lung CD8þ T lympho- fancy was more associated with a subsequent asthma di-
cytes, leading to increased viral replication. Impairment of agnosis between 3 and 5 years of life than RSV bronchiolitis
cytotoxic CD8þ T cells and decreased levels of interferon g in infancy. In children ages 2 to 17 years hospitalized with
provide potential avenues for further investigation of MPV asthma, MPV was the second most commonly (5%) iso-
and other viruses’ ability to hijack the host immune system. lated viral pathogen after rhinovirus.
Further, this T-cell impairment may contribute to the abil- Other clinical symptoms of MPV infection may in-
ity of the virus to reinfect individuals despite preexisting clude nonrespiratory manifestations. In some studies,
antibodies. up to half of all children with MPV had vomiting and/
or diarrhea. Between 5% and 10% of children develop
a rash during the infection. Febrile seizures have been re-
Clinical Manifestations ported uncommonly in patients with MPV infection.
MPV causes both upper and lower respiratory tract dis- MPV has been isolated rarely by PCR from nasal washes
ease. Rhinorrhea and coryza are common, whereas laryn- of children with encephalitis, and one report detected
gitis and croup are described but are less frequent. MPV in the cerebrospinal fluid of a child with encepha-
Children also experience dysphagia associated with de- litis. Identification of MPV outside the respiratory tract is
creased oral intake, and pharyngitis has been associated rare, and children are generally not thought to be viremic
with more than 40% of cases in studies that evaluated during infection.

560 Pediatrics in Review Vol.34 No.12 December 2013


infectious diseases metapneumovirus

Comparison of general clinical and epidemiologic features of


Table 1.

MPV- and RSV-associated lower respiratory tract infection.


Feature MPV RSV
Peak age Older infant (6-12 months) Young infant (<6 months)
Sex Slight male predominance Slight male predominance
Seasonality February-April November-January
Clinical symptoms Fever, rhinorrhea, cough, wheezing, vomiting, Fever, rhinorrhea, cough, wheezing, vomiting,
and diarrhea and diarrhea
Radiographic Consolidation, perihilar infiltrates, atelectasis, Peribronchial thickening, consolidation,
findings hyperinflation atelectasis, hyperinflation
Diagnosis Pneumonia Bronchiolitis
Bronchiolitis Pneumonia
Asthma exacerbation Asthma exacerbation
Risk factors Prematurity, underlying comorbidity, young age Prematurity, young age, underlying comorbidity,
daycare
MPV¼metapneumovirus; RSV¼respiratory syncytial virus.

High-Risk Patients infants without medical problems. A prospective study


Premature infants have increased morbidity and mortality in Argentina following up high-risk and premature in-
from MPV infection. Multiple studies have demonstrated fants noted that 30% of children with MPV infection
that premature and high-risk infants are more likely to be had moderate to severe disease. Other epidemiologic
hospitalized with MPV infection when compared with studies have noted that between 34% and 88% of children

Figure 1. A. Portable chest radiograph of a 5-month-old girl with fever, cough, and increased work of breathing. The lungs are mildly
hyperexpanded, with perihilar interstitial prominence, peribronchial cuffing, and fine alveolar perihilar opacities. There is a confluent
density seen in the right mid lobe, silhouetting the heart border. B. Portable chest radiograph of a 4-month-old boy with fever, cough,
and increased work of breathing. The lungs are hyperinflated, with multifocal atelectasis affecting the right upper lobe and medial lung
bases bilaterally. The airspace opacities affecting the right upper lobe are more confluent compared with elsewhere.

Pediatrics in Review Vol.34 No.12 December 2013 561


infectious diseases metapneumovirus

hospitalized with MPV infection had underlying medical present with sepsis and can develop pulmonary hemor-
conditions, including prematurity, cardiopulmonary dis- rhage and/or acute respiratory distress syndrome. One
ease, or immunodeficiency. In a South African study, human study demonstrated that higher MPV viral loads were
immunodeficiency virus (HIV)–positive children were sig- associated with increased disease severity as manifested
nificantly more likely than HIV-negative children to be in- by fever, prolonged hospital stay, and increased bron-
fected with MPV. chodilator use.
In an adult population, MPV was identified in 3% of Daycare does not seem to be as significant a risk factor
bronchoalveolar lavage specimens in patients who under- for MPV infection compared with other respiratory viruses.
went hematopoietic stem cell transplantation and had Prospective studies of children in daycare demonstrate
lower respiratory tract symptoms. Of these patients, 80% a lower transmission rate of MPV compared with RSV
of the patients died of respiratory failure, indicating that and PIV. Other prospective studies in daycare identified
in a susceptible population MPV can cause high mortality. MPV infection in approximately 10% of all children; how-
Therefore, the host immune system may play a significant ever, this accounted for 2% of all acute respiratory tract
role not only in clearing the infection but also in disease infections.
morbidity.
In the pediatric population, prolonged shedding has Diagnosis
been observed in solid organ transplant recipients. Fur- MPV was initially identified in cell culture; LLC-MK2
thermore, active infection was associated with acute graft monkey kidney cells are commonly used for the growth
rejection. Mortality has been reported in pediatric patients of MPV, but viral cultures take up to 10 to 14 days and
with leukemia, including one patient who was infected are, therefore, not useful clinically. MPV produces small
with 2 genetically different strains of MPV during 10 round plaques with occasional syncytia and can take be-
months and died of acute respiratory distress syndrome tween 3 and 23 days to produce a cytopathic effect (Fig
during the second MPV infection. Another case report de- 2). Shell vial culture has demonstrated increased sensitiv-
tailed a premature infant who required extracorporeal ity compared with traditional culture. Direct and indirect
membrane oxygenation due to MPV-associated acute re- fluorescent antibody testing has similar sensitivity and
spiratory distress syndrome. specificity to shell vial culture. Currently, the gold stan-
Although the predominance of morbidity and mortality dard for diagnosis is PCR testing. Reverse transcriptase–
occurs in high-risk children, case reports describe previously PCR testing has a high sensitivity for virus detection,
healthy children without underlying immunodeficiency and laboratories have developed primers targeted at many
succumbing to MPV infection. These children typically of the conserved proteins. In many clinical laboratories,

Figure 2. Cytopathic effect caused by human metapneumovirus (MPV). A. Uninfected LLC-MK2 cell monolayer. B. MPV-infected
LLC-MK2 cells. Arrows indicate syncytia.

562 Pediatrics in Review Vol.34 No.12 December 2013


infectious diseases metapneumovirus

MPV has been incorporated into multiplex diagnostic be an effective vaccine strategy. Other strategies for pre-
PCR assays used to simultaneously evaluate for multiple vention include the generation of monoclonal antibodies
respiratory pathogens. that could potentially be used as prophylaxis in high-risk
populations.
Treatment
No Food and Drug Administration–approved antiviral
drug against MPV exists. Supplementary oxygen and as-
sisted ventilation may be needed in the hospitalized set- Summary
ting. Intravenous fluids can be used for hydration when
vomiting and diarrhea occur or a patient is unable to tol- • On the basis of strong research evidence and
consensus, (1) (2) (3) (4) human metapneumovirus is
erate oral hydration because of tachypnea or dyspnea. a leading cause of upper and lower respiratory tract
Bronchodilators and steroids may be used in the manage- infections in children.
ment of MPV contributing to asthma or chronic obstruc- • On the basis of research evidence and consensus, (3)
tive pulmonary disease exacerbations, and antibiotics may (5) (6) (7) the clinical features of MPV-associated
be needed in cases of bacterial superinfection, such as disease are similar to those of RSV. MPV is an
important cause of asthma exacerbations,
acute otitis media or suspected community-acquired bac- bronchiolitis, and pneumonia. Bacterial
terial pneumonia. superinfection can occur.
In vitro data suggest that ribavirin and intravenous im- • On the basis of research evidence and consensus, (2)
munoglobulin inhibit MPV infection. Ribavirin has been (3) (8) the mean age of infection is 6 to 12 months,
found to decrease inflammation in a mouse model. Riba- and nearly all school-age children are seropositive.
However, infection can recur, likely in part due to
virin and intravenous immunoglobulin have been used impaired CD8D T-cell response.
together to treat immunocompromised adults and chil- • On the basis of research evidence and consensus,
dren in isolated case reports. However, data are limited, (9) (10) (11) morbidity and mortality are the
and no case-control studies have been performed. In an- highest in patients who are premature, are
imal models, monoclonal antibodies have been effective immunosuppressed, or have underlying
cardiopulmonary abnormalities.
prophylactically and therapeutically at decreasing viral ti- • On the basis of research evidence and consensus, (2)
ter, although no data are available in humans. commercially available diagnostic tests exist, and
reverse transcriptase–PCR is the most commonly
used.
Prevention • On the basis of consensus, because of a lack of
Similar to other respiratory viruses, good hand hygiene relevant clinical studies, recombinant virus vaccines
and curtailing respiratory secretions are currently the only and monoclonal antibodies may be useful as
preventive measures. However, vaccine discovery efforts prophylactics or therapeutics. (10)
are under way. The fusion protein is immunogenic and
highly conserved, making it an excellent target for vac-
cine research. Soluble fusion protein vaccines reduce viral
titers in animal models, and vectored vaccines encoding References
the fusion protein are protective. 1. Edwards KM, Zhu Y, Griffin MR, et al; New Vaccine Surveil-
lance Network. Burden of human metapneumovirus infection in
Researchers have generated live-attenuated temperature- young children. N Engl J Med. 2013;368(7):633–643
sensitive strains of MPV, which produce low levels of 2. van den Hoogen BG, de Jong JC, Groen J, et al. A newly
replication in the upper respiratory tract and no active dis- discovered human pneumovirus isolated from young children with
ease in the lower respiratory tract. High antibody titers respiratory tract disease. Nat Med. 2001;7(6):719–724
were produced, and subsequent infection with wild-type 3. Williams JV, Harris PA, Tollefson SJ, et al. Human meta-
pneumovirus and lower respiratory tract disease in otherwise
virus in a hamster model demonstrated no lower respira- healthy infants and children. N Engl J Med. 2004;350(5):
tory tract infection and decreased viral replication in the 443–450
upper respiratory tract. Thus, cold-passaged MPV may 4. Widmer K, Zhu Y, Williams JV, Griffin MR, Edwards KM,
prove to be a useful vaccine approach. Other recombi- Talbot HK. Rates of hospitalizations for respiratory syncytial virus,
human metapneumovirus, and influenza virus in older adults.
nant strains that lack the small hydrophobic protein
J Infect Dis. 2012;206(1):56–62
and glycoprotein are attenuated and demonstrate evi- 5. Jartti T, van den Hoogen B, Garofalo RP, Osterhaus AD,
dence of subsequent reduction in viral titers on wild-type Ruuskanen O. Metapneumovirus and acute wheezing in children.
infection. Therefore, recombinant viruses may prove to Lancet. 2002;360(9343):1393–1394

Pediatrics in Review Vol.34 No.12 December 2013 563


infectious diseases metapneumovirus

6. Madhi SA, Ludewick H, Kuwanda L, et al. Pneumococcal 9. Englund JA, Boeckh M, Kuypers J, et al. Brief communication:
coinfection with human metapneumovirus. J Infect Dis. 2006;193 fatal human metapneumovirus infection in stem-cell transplant
(9):1236–1243 recipients. Ann Intern Med. 2006;144(5):344–349
7. Williams JV, Tollefson SJ, Heymann PW, Carper HT, Patrie J, 10. Schildgen V, van den Hoogen B, Fouchier R, et al. Human
Crowe JE. Human metapneumovirus infection in children hospital- Metapneumovirus: lessons learned over the first decade. Clin
ized for wheezing. J Allergy Clin Immunol. 2005;115(6):1311–1312 Microbiol Rev. 2011;24(4):734–754
8. Erickson JJ, Gilchuk P, Hastings AK, et al. Viral acute lower 11. Pelletier G, Déry P, Abed Y, Boivin G. Respiratory tract
respiratory infections impair CD8þ T cells through PD-1. J Clin reinfections by the new human Metapneumovirus in an immuno-
Invest. 2012;122(8):2967–2982 compromised child. Emerg Infect Dis. 2002;8(9):976–978

PIR Quiz
This quiz is available online at http://pedsinreview.org. NOTE: Learners can take Pediatrics in Review quizzes and claim credit online only. No paper
answer form will be printed in the journal.

New Minimum Performance Level Requirements


Per the 2010 revision of the American Medical Association (AMA) Physician’s Recognition Award (PRA) and credit system, a minimum performance
level must be established on enduring material and journal-based CME activities that are certified for AMA PRA Category 1 CreditTM. To successfully
complete 2013 Pediatrics in Review articles for AMA PRA Category 1 CreditTM, learners must demonstrate a minimum performance level of 60% or
higher on this assessment, which measures achievement of the educational purpose and/or objectives of this activity.
In Pediatrics in Review, AMA PRA Category 1 CreditTM may be claimed only if 60% or more of the questions are answered correctly. If you score less
than 60% on the assessment, you will be given additional opportunities to answer questions until an overall 60% or greater score is achieved.

1. A 12-month-old boy is admitted to the hospital with rhinorrhea, cough, wheezing, and respiratory distress. The
results of studies on nasopharyngeal fluid wash for respiratory syncytial virus, influenza, and parainfluenza are
negative. Which of the following laboratory studies on nasopharyngeal fluid wash should be ordered to
diagnose the most likely etiologic agent?
A. Direct fluorescent antibody testing.
B. Indirect fluorescent antibody testing.
C. Reverse transcriptase–polymerase chain reaction testing.
D. Shell viral culture.
E. Viral culture.

2. Acute otitis media (AOM) is a common complication of metapneumovirus (MPV) respiratory infection in
children. Research data regarding MPV and increased risk of bacterial coinfection are consistent clinically with
which of the following pathogens identified in children with MPV-associated AOM?
A. Anaerobic bacteria.
B. Haemophilus influenzae.
C. Moraxella catarrhalis.
D. Streptococcus pneumoniae.
E. Streptococcus pyogenes.

3. A 2-year-old child presents in early April with fever, rhinorrhea, and coryza. A viral cause is suspected for this
child’s illness. Which of the following additional signs or symptoms is most likely to indicate that MPV is the
etiologic agent?
A. Conjunctivitis.
B. Cough.
C. Croup.
D. Laryngitis.
E. Pharyngitis.

564 Pediatrics in Review Vol.34 No.12 December 2013


infectious diseases metapneumovirus

4. In which group of pediatric patients has prolonged viral shedding of MPV been observed?
A. Chronic pulmonary disease patients.
B. Congenital heart disease patients.
C. Human immunodeficiency virus–positive patients.
D. Premature infants.
E. Solid organ transplant recipients.

5. A few pediatric patients with severe morbidity and mortality secondary to MPV disease are previously healthy
children without underlying disorders. Which of the following is a typical presentation for this group of
children?
A. Encephalitis.
B. Pneumonia.
C. Seizures.
D. Sepsis.
E. Vomiting, diarrhea, and dehydration.

Corrections
In the print version of the November 2013 article “Cephem Antibiotics: Wise Use Today Preserves Cure for Tomorrow”
(Parker S, Mitchell M, Child J. Pediatrics in Review. 2013(34);11:510–524, doi: 10.1542/pir.34-11-510), in the Selected
References introduction, the link to the complete reference list should be: http://pedsinreview.aappublications.org/
content/34/11/510/suppl/DCSupplementary_Data. The link is correct in the online version of the journal. The journal regrets
the error.
In the print version of the November 2013 article “Chronic Cough in Children: a Primary Care and Subspecialty Collab-
orative Approach” (Kaslovsky R and Sadof M. Pediatrics in Review. 2013;34(11):498–509, doi: 10.1542/pir.34-11-498), the
following phrase appeared above the quiz questions but should have been part of Question 1: “Some clinical signs are highly
suggestive of a given condition. In the scenarios below, match the clinical presentation with the most likely cause of the
patient’s cough.” The phrase appears correctly in the online version of the journal CME quiz. The journal regrets the error.

Pediatrics in Review Vol.34 No.12 December 2013 565


index of suspicion

Case 1: Swelling Behind the Knee in a 15-Year-Old


Boy
Case 2: Fatigue, Weight Loss, Fever, and Atypical Skin
Lesion in an Adolescent Boy
Case 3: New-Onset Murmur and Signs of Shock in
a 15-Year-Old Girl
Case 1 Presentation weight is 71 kg, height is 177 cm, tem-
The reader is encouraged to write A 15-year-old boy presents to the pe- perature is 37.4°C, heart rate is 102
possible diagnoses for each case before diatric clinic with a 3-week history of beats per minute, respiratory rate is
turning to the discussion. a mass behind his right knee. He 16 breaths per minute, and blood pres-
found it incidentally while bathing sure is 113/67 mm Hg. There is a 3 
and experiences no pain or discomfort 2-cm, raised lesion that is erythematous
The reader is encouraged to write
in his right knee during sports activi- with a violaceous hue and areas of
ties. He denies a history of preceding crusting and scaling on the upper me-
possible diagnoses for each case
trauma. His medical history, including dial aspect of the right thigh (Fig 2).
before turning to the discussion. We personal history and family history, He has a 2-cm, single, firm, nontender,
invite readers to contribute case are negative for inflammatory arthritis mobile lymph node in the right ingui-
presentations and discussions. Please and malignant tumor. nal region. The rest of the findings on
inquire first by contacting Dr Deepak Physical examination reveals a teen- physical examination are normal, with
Kamat at DKamat@med.wayne.edu. age boy with an ovoid, nontender, mo- no other adenopathy, hepatospleno-
bile mass with a cystic feeling in the megaly, or bone tenderness.
posteromedial aspect of the right knee Laboratory examination reveals
(Fig 1). The mass is approximately the following: white blood cell count,
Author Disclosure
9.8  3  2 cm, firm, and noncompress- 9200/mL (9.2  109/L), with 75%
Drs Sankararaman, Patra, Wells, ible. The swelling becomes prominent neutrophils, 12% lymphocytes, 11%
Sarker, Kitchen, Jasty, Benza, and on extension of the knee. The swell- monocytes, 1% eosinophils, and 1%
Sahai have disclosed no financial ing is also brilliantly transilluminant. basophils; hemoglobin, 13 g/dL
relationships relevant to this article. The rest of the examination findings (130 g/L); platelet count, 439 
are within normal limits. A radiologic 10 3 /mL (439  109/L); normal pe-
This commentary does not contain
test confirms the physical finding. ripheral smear; erythrocyte sedimen-
discussion of unapproved/
tation rate (ESR), 103 mm/h; and
investigative use of a commercial C-reactive protein (CRP), 1.0 mg/L
product/device. Case 2 Presentation (9.5 nmol/L) (reference range,
A 15-year-old previously healthy boy 0-0.08 mg/dL [0-0.76 nmol/L]).
presents with a 2-month history of Results of a comprehensive metabolic
a red skin lesion on the right upper panel, as well as serum lactic dehydro-
thigh that has progressively increased genase and uric acid levels, are normal.
in size. He also has a 3-week history Further testing and a biopsy specimen
of fatigue, decreased appetite, weight from the enlarged right inguinal lymph
loss of 7.2 kg, worsening lower back node lead to the diagnosis.
pain, and a painful lump in the right
groin. For the last 3 to 4 days, he has
had fevers and night sweats. There is Case 3 Presentation
a history of contact with 2 indoor cats A 15-year-old African American girl
and travel to Mexico as a tourist 9 presents with fever (maximum temper-
months before this visit. ature of 39.2°C), sore throat, decreased
Physical examination reveals a pale oral intake, headache, vomiting, and
and ill-looking adolescent boy. His dizziness of 2- to 3-day duration. She

566 Pediatrics in Review Vol.34 No.12 December 2013


index of suspicion

have pharyngeal erythema. Her pe- is prescribed ceftriaxone and vanco-


ripheral pulses are hyperdynamic, mycin and admitted with the diagno-
and she has 2/6 harsh holosystolic sis of sepsis. Additional tests reveal the
murmur, which is loudest at apex. diagnosis.
Findings on the rest of the physical
examination are normal.
Laboratory results are as follows:
white blood cell count, 18,600/mL Case 1 Discussion
A radiograph of the knee did not
(18.6  109/L), with 75% neutro-
reveal any abnormality. However,
phils, 14% lymphocytes, and 10%
ultrasonography demonstrated a well-
monocytes; hemoglobin, 10.5 g/dL
circumscribed hypoechoic cystic swell-
(105 g/L); and platelet count,
ing in the posteromedial aspect of the
553  103/mL (553  109/L). Elec-
right knee without joint effusion (Figs
trocardiography reveals normal sinus
Figure 1. Clinical photograph showing 3 and 4). A popliteal cyst was diag-
rhythm. The CRP level is 12.0 mg/L
a popliteal cyst in the posteromedial nosed in view of the benign nature
(114.3 nmol/L), ESR is 109 mm/h,
aspect of the right knee (white arrows). of this cystic swelling.
serum electrolytes are normal, and
ASO titer is 160 U/L. Urine analysis
also experiences breathlessness, which The Condition
reveals the following: specific gravity, A popliteal cyst is a synovial fluid–
has not been responding to multiple in- 1.031; pH 7; protein, 1þ; urobilino-
haled albuterol treatments at home. filled mass located in the popliteal
gen, 4þ; ketones, 3þ; nitrites, posi- fossa. It is commonly seen in both
On examination, her pulse is 128 tive; LE, 2þ; white blood cells, 5 to
beats per minute, respiratory rate is adults and children, although only
10 per high-power field; and red approximately one-third of popliteal
30 breaths per minute, temperature blood cells, less than 2 per high-power cysts are reported in children. Popli-
is 38.5°C, and blood pressure is field. The results of chest and abdom- teal cysts are also referred to as Baker
90/55 mm Hg in both arms. Her inal radiography are normal. Echocar- cysts, named after William Morrant
weight is 72.70 kg (>90th percen- diography reveals flow acceleration Baker, who described it in 1877.
tile), and her height 183.00 cm across the left pulmonary artery with Popliteal cysts in children are mostly
(99th percentile). She is found to a peak gradient of 31 mm Hg. She idiopathic. The occurrence in boys out-
numbers the occurrence in girls in a
ratio of 2:1, and most cases occur uni-
laterally. The patients are often asymp-
tomatic, and most cases are detected
incidentally by the patient, parent, or
physician as a painless mass behind
the knee. In one study, the prevalence
of asymptomatic popliteal cysts in chil-
dren detected by ultrasonography was
reported as 2.4%. The mass gradually
increases in size and may be fairly large
when the patient first notices it. Physi-
cal examination reveals a firm cystic
mass located in the medial aspect of
the popliteal fossa, usually distal to
the popliteal crease. It also becomes
prominent on extension of the knee
joint. Positive transillumination is a use-
ful bedside diagnostic test. Secondary
Figure 2. A 3 3 2-cm, raised erythematous lesion with a violaceous hue and areas of popliteal cysts are commonly encoun-
crusting and scaling on the upper medial aspect of the right thigh. tered in children with underlying

Pediatrics in Review Vol.34 No.12 December 2013 567


index of suspicion

is seldom required for diagnosis. In


some instances, MRI may be required
if the ultrasonogram is equivocal in de-
lineating cystic mass from solid tumors.
The presence of a solid mass warrants
urgent referral to a specialist for further
evaluation, possible biopsy, and appro-
priate treatment (including excision,
chemotherapy, and/or radiotherapy).
MRI is also recommended for further
evaluation if there is clinical suspicion
for underlying joint conditions due to
persistent knee pain, joint swelling, or
systemic symptoms.

Management
Most authors report a spontaneous
resolution of idiopathic popliteal cysts
within a period of 20 months. Hence,
conservative management by watchful
Figure 3. Ultrasonogram of the right knee (posterior sagittal view) showing the waiting is strongly recommended in
hypoechoic cystic mass (white arrows) with internal debris. children. On the contrary, in adults,
treatment is directed toward intra-
joint disorders that cause knee ef- Normal debris inside the cyst can be articular disease and recurrence of pop-
fusions, such as juvenile idiopathic seen as echogenic internal signals on liteal cyst is common. Rupture of the
arthritis, septic arthritis, psoriatic ar- ultrasonography (Fig 3). In a recent cyst, hemorrhage, infection, or malig-
thritis, meniscal tears, hemophilia, study in children, the presence of nant transformation can occur rarely
pigmented villonodular synovitis, or these echogenic internal signals was in children, so new or increased pain
unstable osteochondritis dissecans. frequently encountered in patients should warrant immediate evaluation.
The common mean age of presen- with arthritis and further investigation Surgical treatment is indicated in chil-
tation of idiopathic popliteal cysts is with magnetic resonance imaging dren only in the presence of symptoms
6 years, with a range of 2 to 14 years. (MRI) was recommended. (pain and/or increasing size that limits
The most common site of origin is the motion).
bursa of the gastrocnemius and semi- Differential Diagnosis Our patient is older than the typ-
membranosus muscles. Another com- Soft tissue tumors, such as lipoma, ical patient with idiopathic popliteal
mon site of origin is herniation of the xanthoma, hemangioma, fibrosarcoma, cysts, and ultrasonography also revealed
synovium through the posterior joint synovial cell sarcoma, and tumors of echogenic internal signals. Hence,
capsule of the knee. In adults, pop- the tendon sheath, should be in- we recommended further evaluation
liteal cysts are usually secondary to cluded in the differential diagnosis. with MRI for the presence of under-
osteoarthritis, meniscal tears, and in- In older children and adults, a rup- lying joint diseases. However, the pa-
flammatory joint conditions. Unlike tured popliteal cyst may mimic deep tient’s family decided to wait because
in adults, the presence of underlying vein thrombosis or thrombophlebitis. he is mostly asymptomatic. He par-
intra-articular disease is rare in chil- Knee radiographs should be obtained ticipates in basketball without any
dren. However children with knee to evaluate bony lesions, such as osteo- pain or discomfort and is under
pain or joint swelling should undergo chondromas, osteochondritis dissecans, follow-up.
evaluation for an underlying cause, and malignant bone tumors. The diag-
such as septic arthritis, Lyme disease nosis of a popliteal cyst is confirmed by
Lessons for the Clinician
(for patients living in, or visiting, en- ultrasonography, which differentiates
demic areas in the last year), inflam- a solid mass from a hypoechoic cystic • Baker cysts or popliteal cysts are
matory arthritis, or structural injury. lesion. Aspiration of the cyst in children common in children.

568 Pediatrics in Review Vol.34 No.12 December 2013


index of suspicion

and soft tissue density in the right inner


thigh.
An excisional biopsy specimen of
the right inguinal lymph node revealed
complete replacement by a large cell
neoplasm with a moderate amount
of amphophilic cytoplasm containing
generally a single round to slightly
spindled nucleus that is pleomorphic
with clumped chromatin and promi-
nent nucleoli. Whole-body positron
emission tomography (PET)–CT re-
vealed extensive fludeoxyglucose-avid
adenopathy in the right inguinal re-
gion, in the right internal and external
iliac chain, throughout the retroperi-
toneum, in the right hilum, in the su-
perior anterior mediastinum, and in
the base of the left side of the neck.
Bone marrow biopsy and cerebrospi-
nal fluid test results were negative for
malignant neoplasms.
Immunoperoxidase staining was
positive for CD30 (Ki-1), anaplastic
lymphoma kinase 1 (ALK-1), BCL6,
and CD43 and weakly positive for
Figure 4. Ultrasonogram (posterior transverse view) showing the relationship of the epithelial membrane antigen. Ap-
popliteal cyst (black and white arrows) with the knee joint. proximately 60% to 70% of nuclei
were positive for Ki-67. The patient
was diagnosed as having ALK-positive
• Popliteal cysts are mostly asymp-
stage 3 anaplastic large cell lymphoma
tomatic and incidentally found as Case 2 Discussion (ALCL).
painless masses behind the knee.
The results of serologic tests for an in-
• In children, most cases are idio-
fectious cause of lymphadenitis, includ- The Condition
pathic and resolve spontaneously,
ing Bartonella henselae, histoplasmosis, Non-Hodgkin lymphoma represents
so surgical excision is not required.
blastomycosis, coccidiomycosis, asper- approximately 60% of all lymphomas
• Ultrasonography confirms the cys-
gillosis, syphilis, and human immuno- in the pediatric population. ALCL
tic nature of the swelling, which is
deficiency virus, were negative, and a accounts for 10% to 15% of all child-
also helpful in the evaluation of the
tuberculin skin test result was nonreac- hood non-Hodgkin lymphomas and
characteristics of the swelling and
tive. Evaluation for vasculitis included is typically associated with a T-cell
also reveals the presence or ab-
a proteinase 3 antineutrophil cyto- (65%) or null cell (35%) phenotype.
sence of the joint effusion.
plasmic antibody and myeloperoxidase There are 2 clinical presentations of
• On the basis of clinical or radio-
antineutrophil cytoplasmic antibody, childhood ALCL. One is a primary
logic suspicion, MRI is recom-
and both results were negative. Com- cutaneous form that can present as
mended for further evaluation of
puted tomography (CT) of the neck, a single or multiple cutaneous
underlying joint disorders in sec-
chest, abdomen, and pelvis revealed lesion(s). This presentation is rare in
ondary popliteal cysts.
lymphadenopathy in the upper medias- pediatric patients but when present
(Senthilkumar Sankararaman, MD, tinum (largest diameter, 1 cm), retro- poses a diagnostic challenge in distin-
Kamakshya Patra, MD, and Wanda peritoneal region (largest diameter, guishing it from systemic ALCL with
Wells, MD, Louisiana State University 1.4 cm), right inguinal region (largest secondary cutaneous involvement. In
Health Sciences Center, Shreveport, LA) diameter, 2 cm), and a focal area of skin the primary systemic form of ALCL

Pediatrics in Review Vol.34 No.12 December 2013 569


index of suspicion

more than 90% of patients present The staging of ALCL is based on direct the clinician to search
with nodal disease. However, a high the St Jude system modified from for a malignant tumor, such as
incidence of extranodal involvement Murphy. Stages 1 and 2 are consid- lymphoma.
(involving the skin in 25%, bone ered localized disease, and stages 3 • Primary cutaneous ALCL is rare in
in 17%, lung in 10%, and liver in and 4 are disseminated disease. Bone children. Even in the absence of
8%) has been reported previously. marrow involvement in ALCL, if constitutional symptoms, patients
Central nervous system or bone mar- present, carries a worse prognosis. presenting with skin lesions that
row involvement is rare with this ALK positivity in the primary sys- are diagnostic of ALCL should
lymphoma. Approximately 75% of temic form, as is seen in our patient, have a thorough evaluation, in-
children with ALCL can present with is associated with a better prognosis, cluding CT of the neck, chest, ab-
B symptoms described as tempera- whereas ALK negativity carries a poor domen, and pelvis, and a PET scan
tures greater than 38°C for 3 consec- prognosis. to rule out systemic ALCL with
utive days, night sweats, or unexplained secondary skin involvement be-
weight loss of more than 10% of body cause management is more aggres-
weight in the prior 6 months. (In the Management sive in the latter.
Ann Arbor staging of lymphomas, A Primary cutaneous forms of ALCL
typically are treated with surgical ex- (Tania Sarker, MD, MPH, Univer-
indicates absence of systemic symp-
cision or localized radiotherapy. The sity of Toledo College of Medicine,
toms, whereas B indicates their pres-
systemic form of ALCL is treated Toledo, OH, and Brenda Kitchen,
ence.) Our patient had the systemic
with combination chemotherapy. Mul- MD, and Rama Jasty, MD, Univer-
form of ALCL with secondary cuta-
tiple treatment regimens have been sity of Michigan Medical School, Ann
neous involvement.
used by various cooperative groups Arbor, MI)
CD30 (Ki-1) is strongly expressed
in most ALCL, although this is not worldwide. Our patient was treated
unique to this lymphoma. Recently, according to Pediatric Oncology
in addition to the CD30 expression, Group trial 9315, which includes Case 3 Discussion
ALK positivity has not only helped doxorubicin, prednisone, and vin- On further questioning it was found
with the diagnosis but has led to cristine with methotrexate given that the patient was diagnosed as hav-
a better understanding of the disease later during consolidation for a total ing uveitis and juvenile idiopathic ar-
at the molecular level. More than treatment duration of 52 weeks. thritis in the past. However, she was
90% of children with systemic ALCL This regimen has a 4-year event-free undergoing no treatment because
are reported to be ALK positive. ALK survival and overall survival of 71.8% she is in remission and asymptomatic.
is a membrane-bound tyrosine kinase and 88.1%, respectively, in advanced The patient’s sister was on a liver
normally expressed in neural cells but stage pediatric ALCL, which is com- transplantation list because of auto-
not in lymphoid tissue. The charac- parable to other regimens with fewer immune hepatitis. Our differential
teristic t(2;5) translocation between associated toxic effects. Using this diagnosis at this time included sys-
chromosomes 2 and 5 (t[2:5] [p23: regimen, our patient went into a com- temic inflammatory response syn-
q35]) is present in 83% of the pedi- plete remission within 3 weeks. drome (SIRS) due to sepsis, lupus,
atric cases with systemic ALCL. This antiphospholipid syndrome, vasculi-
translocation involves a fusion of tis, bacterial endocarditis, and pyelo-
Lessons for the Clinician
the cytoplasmic domain of the nephritis. To investigate this further,
ALK gene on 2p23 to the NPM • ALCL can mimic a skin lesion of we ordered a rapid streptococcal an-
gene coding for the nuclear phos- infectious origin or vasculitis. tigen test, mononucleosis spot test, Ep-
phoprotein on 5q35. NMP-ALK • Nodal involvement and constitu- stein-Barr virus serologic test, blood
acts as a constitutively active tyro- tional B symptoms (fever, night cultures on 3 different occasions,
sine kinase that can trigger malignant sweats, and weight loss) are thyroid function tests, serum ferritin
transformation or activate antiapop- common features of ALCL in level, autoantibody panel (antinu-
totic pathways. The fluorescent in children. clear antibody, extractable nuclear
situ hybridization probe from the in- • The presence of atypical skin le- antigen, antineutrophil cytoplasmic
guinal lymph node of our patient re- sions and B symptoms even in antibody, and anti–double-stranded
vealed the presence of t(2;5) in 34% the absence of significant periph- DNA), rapid plasma reagin, lupus anti-
of the nuclei. eral lymphadenopathy should coagulant test, serum cytomegalovirus

570 Pediatrics in Review Vol.34 No.12 December 2013


index of suspicion

IgG and IgM antibody titers, and Diagnosis stage as described in the diagnostic
tuberculin test. The results were all In 2010, the adult criteria for TA were criteria developed by EULAR and
negative. adapted to the pediatric population by PRES.
The patient remained febrile and the European League against Rheu- Imaging may include routine radio-
tachycardic, with clinical manifesta- matism (EULAR) and Pediatric Rheu- graphy and Doppler ultrasonography,
tions consistent with SIRS despite matology European Society (PRES). which looks not only at vessel wall
broad-spectrum antibiotics and a TA diagnosis using these criteria re- thickness but also at blood flow.
large volume of intravenous fluid quires typical angiographic abnormal- Conventional angiography is still
support, and her ESR and CRP level ities of the aorta or its main branches the gold standard, but it is an invasive
remained markedly elevated. Dopp- and pulmonary arteries (mandatory procedure, requires injection of con-
ler ultrasonography of her neck criteria) plus 1 of 5 clinical and labora- trast, and exposes the patient to radi-
performed to look for evidence of tory criteria, such as pulse deficit or ation. More specific and sensitive
Lemierre syndrome (infectious throm- claudication, blood pressure discrep- vessel wall imaging by CT, MRI, or
bosis of the jugular vein caused by ancy in any limbs, bruits, systolic magnetic resonance angiography is
Fusobacterium necrophorum in most and diastolic blood pressure above preferred and an essential method
cases, usually seen after a dental infec- the 95th percentile for height, and el- for diagnosing and monitoring the
tion, tonsillitis, or pharyngitis) revealed evated acute phase reactants. (2) disease.
circumferential wall thickening of TA diagnosis is challenging in the
both common carotid arteries. On early prepulseless phase, where there Management
the basis of these findings, MRI of Glucocorticoids are the first line of
is only thickening of intima due to an
the chest was performed, which re- treatment, and if remission is not
inflammatory infiltrate and no evi-
vealed circumferential wall thickening achieved, methotrexate or azathio-
dence of stenosis or aneurysm forma-
and enhancement that involved multi- prine can also be added. Surgical by-
tion. The symptoms if any at this
ple large vessels, including the aorta, pass and percutaneous transluminal
stage are nonspecific and presumed
pulmonary arteries, and the common angioplasty are reserved for situations
to be due to the production or secre-
carotids, suggesting a diagnosis of where arterial stenosis is affecting a vi-
tion of inflammatory cytokines. The
vasculitis, such as Takayasu arteritis tal organ.
symptoms include fever, weight loss,
(TA). For patients who do not respond
fatigue, and myalgias and may be eas-
to immunosuppressive therapy, many
ily mistaken for an infection, as in our
The Condition other immunomodulatory therapies,
The first case of TA was described in patient. Patients may also have hyper-
including anti–tumor necrosis factor
1908 by Dr Mikito Takayasu at the tension, cardiac murmurs, vascular
agents, have been tried. Morbidity
annual meeting of the Japan Oph- bruits, aortic arch dilatation, cardiac
and mortality at a young age are very
thalmology Society as “a case of pe- failure due to a high-output state high; therefore, early recognition and
culiar changes in the central retinal from the aortic regurgitation, and intervention may help improve the
vessels” in a 21-year-old female pa- myocardial dysfunction, such as our outcome.
tient. Today we know that TA is patient displayed. Carotid and verte- Our patient was prescribed high-
the most common large and me- bral artery involvement causes reduc- dose intravenous methylprednisolone
dium vessel vasculitis in children. It tion in cerebral blood flow, leading and then switched to oral prednisone,
is an uncommon condition, which to headaches, vertigo, and syncope. which was tapered. She has respond-
manifests in the first or second de- Ophthalmologic features are amau- ed to this therapy.
cade of life, with a mean age of pre- rosis fugax and retinopathy. Once
sentation of 13 years and reported the disease progresses to the pulseless
stage due to chronic fibrosis and Lessons for the Clinician
female-male ratio ranges from 1.2:1
in Israel to 6.9:1 in Mexico. TA is subsequent vessel wall stenosis, the • Even though sepsis is the most
an idiopathic chronic inflammatory clinical findings are related to the re- common cause of SIRS, autoim-
disease with granulomatous panar- stricted blood flow from the affected mune disorders such as vasculitis
teritis. TA is associated in some arteries. Claudication (stenosis of ab- should be considered in the differ-
countries with tuberculosis infec- dominal aorta), hypertension (stenosis ential diagnosis of patients who
tion, but in most cases the cause is of renal arteries), and blood pressure present with clinical manifestations
unknown. (1) differential are part of the pulseless of SIRS.

Pediatrics in Review Vol.34 No.12 December 2013 571


index of suspicion

• When clinical examination reveals a References matosis and childhood Takayasu arteritis:
new murmur and echocardiography 1. Brunner J, Feldman BM, Tyrrell PN, Ankara 2008, part II: final classification
criteria. Ann Rheum Dis. 2010;69(5):
reveals an increase in flow in the et al. Takayasu arteritis in children and
adolescents. Rheumatology (Oxford). 2010; 798–806
pulmonary artery, it may raise
49(10):1806–1814
suspicion for a vasculitis, such as 2. Ozen S, Pistorio A, Iusan SM, et al;
TA. Paediatric Rheumatology International To view Suggested Reading lists
Trials Organisation (PRINTO). EULAR/
(Natalia Benza, MD, and Shashi PRINTO/PRES criteria for Henoch-
for these cases, visit http://pedsinreview.
Sahai, MD, Children’s Hospital of Schönlein purpura, childhood polyarteritis aappublications.org and click on the
Michigan, Detroit, MI) nodosa, childhood Wegener granulo- “Index of Suspicion” link.

Parent Resources From the AAP at HealthyChildren.org


Case 2
• English: http://www.healthychildren.org/English/health-issues/conditions/cancer/Pages/Symptoms-of-Childhood-
Cancers.aspx
• Spanish: http://www.healthychildren.org/spanish/health-issues/conditions/cancer/paginas/symptoms-of-childhood-
cancers.aspx

572 Pediatrics in Review Vol.34 No.12 December 2013


in brief

In Brief
Down Syndrome
Wendy L. Hobson-Rohrer, MD, MSPH maternal age–related risk (whichever is acute and serous otitis media, sinusitis,
Lisa Samson-Fang, MD higher). Yet, the risk of having another and vision problems. Because 40% to
University of Utah, Salt Lake City, UT child with DS is greater for a young 50% of children with DS may have a
woman who is a carrier of a balanced cardiac anomaly, all infants should un-
translocation than for a middle-aged dergo screening echocardiography.
Author Disclosure woman. It is critical that all children Clinicians should evaluate infants
Drs Hobson-Rohrer and Samson-Fang born with DS have chromosome anal- with any sign that may suggest a gastro-
have disclosed no financial ysis to identify those having a translo- intestinal malformation because these
relationships relevant to this article. cation and ensure accurate genetic conditions comprise the second most com-
counseling for the family. mon major area of congenital anomaly
This commentary does not contain
Screening for DS should be offered occurring in DS and include duode-
discussion of unapproved/ to all pregnant women. Prior screening nal atresia, tracheoesophageal fistula,
investigative use of a commercial guidelines identified 50% of infants in Hirschsprung disease, and imperforate
product/device. utero, whereas new screening guide- anus. By adulthood, up to 75% of indi-
lines have first trimester detection rates viduals with DS will develop hearing
of 82% to 85% and second trimester problems, 15% will develop cataracts,
Health Supervision for Children With rates of 80%; combination screening 50% to 75% will develop obstructive
Down Syndrome. American Academy during both trimesters identifies 95% sleep apnea, and 5% will develop celiac
of Pediatrics Committee on Genetics. of cases. When an infant is born with disease. Up to 13% of children with DS
Pediatrics. 2011;128(2):393–406 DS, parents wish to be congratulated may develop a seizure disorder.
Updated National Birth Prevalence on the birth and then receive disclosure Autoimmune thyroid disease in-
Estimates for Selected Birth Defects in of the diagnosis in a sensitive manner. creases in prevalence with age, and
the United States, 2004-2006. Parker
Principles of sharing the difficult news 60% develop hypothyroidism by adult-
SE, Mai CT, Canfield MA, et al; for the
aspects of the infant’s condition appro- hood. Children with DS tend to have
National Birth Defects Prevention
Network. Birth Defects Res A Clin Mol priately include disclosing concern as weaker immune systems and should be
Teratol. 2010;88(12):1008–1016 soon as suspicion exists, telling both monitored closely when ill. The 2011
Down Syndrome. In: Jones K. Smith’s parents at the same time, and explain- updated guidelines published by the
Recognizable Patterns of Human ing the range of variability of presen- American Academy of Pediatrics should
Malformation. 5th ed. Philadelphia, tations and the potential associated be followed, which recommend annual
PA: Saunders; 1997:8–13 medical conditions. screening for many of the above condi-
Current Dilemmas in Down Syndrome Physical features of DS are variable tions and periodic surveillance for others.
Clinical Care: Celiac Disease, Thyroid and include hypotonia, epicanthal folds, Hematologic problems are also rela-
Disorders, and Atlantoaxial Instability. flat nasal bridge, slanted palpebral fis- tively common in children born with DS.
Cohen WI. Am J Med Genet C Semin
sures, speckling of the iris, abnormal Ten percent will have a transient mye-
Med Genet. 2006;142C(3):141–148
auricles, hyperflexibility, excessive skin- loproliferative disorder at birth, whereas
folds in the posterior aspect of the neck, 1% of patients with DS will develop
Down syndrome (DS) is one of the most a single transverse palmar crease, and leukemia during a lifetime. Congenital
common genetic conditions. Incidence a wide gap between the first and sec- myeloproliferative conditions generally
of DS is 1 in 691 births but varies greatly ond toes. resolve but are a risk factor for later de-
with maternal age (eg, for women age Certain medical problems occur with velopment of leukemia. Guidelines rec-
35–39 years, the incidence increases to a high prevalence in children born with ommend a complete blood cell count at
1 in 270). Most cases of DS are due to DS, forming the basis for the recom- birth. If a newborn has a normal blood
sporadic mutations that result in an extra mendations for the health supervision cell count, no further monitoring is rec-
chromosome 21. Generally, the recur- of these children. Many common ill- ommended; however, if the child man-
rence risk is approximately 1% or the nesses occur more frequently, including ifests signs such as petechiae, weight

Pediatrics in Review Vol.34 No.12 December 2013 573


in brief

loss, pallor, or fevers, a complete blood cervical spine must be protected and diac and gastrointestinal malformations.
cell count should be obtained. Guide- that excessive extension or flexion dur- Parents and families benefit from the
lines also recommend yearly monitoring ing anesthetic, surgical, or radiographic support of a skilled primary care phy-
of hemoglobin concentration (with a procedures must be avoided. The clini- sician who provides evidence-based
ferritin level and C-reactive protein cian should also perform a neurologic health anticipatory guidance and assists
measurement) if the child has any risk examination to monitor for emergence them in understanding the developmen-
factors for iron deficiency. of increased tone or hyperreflexia. Chil- tal abilities of the child while establish-
Although earlier 2001 American dren with DS should avoid sports such ing realistic expectations. Parents value
Academy of Pediatrics health supervi- as soccer, football, gymnastics, and having access to other parents who have
sion guidelines recommended cervical trampoline use. Parents should be ad- children with DS within a practice or
spine radiographs performed between vised that if they note changes in gait, through local or national organizations
ages 3 and 5 years in children born arm or hand function, or bowel or blad- and benefit from counseling regarding
with DS, the 2011 guidelines do not. Al- der control, as well as weakness, atyp- the emotional health of siblings and
though children with DS have a 1% to ical head positioning, or chronic neck family members. DS is another example
2% risk of atlantoaxial instability, evi- pain, they should consult their primary of a disease with broad phenotypic dif-
dence does not support routine radio- care physician promptly because further ferences. Transitions to adult medical
graphic screening. However, careful evaluation is warranted. care are imperative to address issues
surveillance is imperative. Because cer- of fertility and contraception, along with
tain sports place children with DS at Comments: DS was first described future employment opportunities and liv-
an increased risk of spinal cord injury, by J. L. H. Down in 1866. Because there ing arrangements as these patients enter
the Special Olympics organization may is great variability in presentation, par- adulthood.
require radiographs for participation. ents must receive a balanced perspec-
At least twice a year, the clinician tive when being counseled. Prenatal Janet R. Serwint, MD
should review with parents that the imaging may help in better defining car- Consulting Editor, In Brief

Parent Resources From the AAP at HealthyChildren.org


• English: http://www.healthychildren.org/English/health-issues/conditions/developmental-disabilities/Pages/Children-
with-Down-Syndrome-Health-Care-Information-for-Families.aspx
• English: http://www.healthychildren.org/English/health-issues/conditions/developmental-disabilities/Pages/Down-
Syndrome.aspx

Answer Key for December 2013 Issue:


Type 2 Diabetes Mellitus: 1. C; 2. A; 3. B; 4. D; 5. E.
Managing Feeding Problems and Feeding Disorders: 1. D; 2. C; 3. B; 4. D; 5. D.
Human Metapneumovirus: 1. C; 2. D; 3. E; 4. E; 5. D.

574 Pediatrics in Review Vol.34 No.12 December 2013


visual diagnosis

An 11-Month-Old With Nausea,


Vomiting, and an Abdominal Mass
Laura A. Whittington, DO,* David C. Stevens, MD,* Sarah
A. Jones, MD,* Julie M. Mayo, DO*†

Presentation
An 11-month-old girl presents to the emergency depart-
ment (ED) with 12 episodes of nonbilious, nonbloody
vomiting and decreased activity for the past 24 hours.
Her medical history and family history are notable for
the following. She was born at term via an uncomplicated,
induced vaginal delivery. There is a paternal history of di-
abetes. The patient’s half-sister, who has phenylketonuria,
was ill with nausea without vomiting 2 days before the pa-
tient’s presentation. Yesterday the patient was seen in the
ED for nausea and 5 bouts of emesis. She was discharged
after receiving one dose of ondansetron and could tolerate
clear liquids by mouth. She now returns to the ED for con-
tinued vomiting.
On physical examination the patient’s temperature is
37.9°C, heart rate is 114 beats per minute, respira-
tory rate is 30 breaths per minute, blood pressure is
105/65 mm Hg, and oxygen saturation is 98% in room
air. She is comfortable and sleeping but awakens when
physically examined. Mucous membranes are slightly dry,
and her posterior oropharynx is erythematous. Cardiac aus-
Figure 1. Abdominal bulge due to underlying mass.
cultation reveals normal S1 and S2 heart sounds without
a murmur. She has good peripheral perfusion and normal
capillary refill. Her lungs are clear to auscultation. She has
Author Disclosure no abdominal scars, and bowel sounds are normal. Her ab-
Drs Whittington, Stevens, Jones, and Mayo have disclosed no domen is soft and nontender, and no masses are palpable.
The spleen and liver are not enlarged. Abdominal radiog-
financial relationships relevant to this article. This
raphy reveals a normal bowel gas pattern.
commentary does not contain discussion of unapproved/ The patient is admitted for intravenous hydration. De-
investigative use of a commercial product/device. spite treatment with ondansetron, frequent vomiting per-
sists. During her second hospital night, she has several
large bowel movements that contain blood and mucus.
Her father notices a visible mass in her abdomen (Fig 1).
Laboratory evaluation reveals that her stool sample is neg-
ative for Shiga-like toxin and rotavirus. An abdominal ra-
diograph is suggestive of underlying disease (Fig 2). On
the basis of the radiographic findings, a water-soluble con-
trast enema is performed (Fig 3) that confirms the sus-
pected underlying diagnosis.

*Department of Pediatrics, University of South Dakota Sanford School of Medicine, Sioux Falls, SD.

Pediatric Hospital Service/Sanford Children’s Hospital, Sioux Falls, SD.

Pediatrics in Review Vol.34 No.12 December 2013 e47


visual diagnosis

Figure 3. Water-soluble contrast enema reveals an intra-


intestinal mass.

between age 4 and 7 months are most commonly affected.


Intussusception rarely occurs in children younger than 2
Figure 2. Abdominal radiograph showing paucity of air in
months, and 70% of cases occur in the first year of life. Most
bowel and a curvilinear mass within the course of the colon.
cases are idiopathic, but some, particularly those occurring
in children outside the normal age range, form around path-
Diagnosis ologic lead points, such as a Meckel diverticulum, intestinal
Intussusception polyp, or edema caused by Henoch-Schönlein purpura. Males
The patient’s history of bloody stools, a palpable abdom- develop intussusception at roughly twice the rate of females.
inal mass, and the soft tissue opacification on abdominal The incidence is estimated to be 18 to 56 per 100,000 live
radiography are highly suggestive of ileocecal intussuscep- births, and there is evidence in the United States and abroad
tion. Subsequent contrast enema revealed a large intussus- that the incidence is decreasing overall.
ception that involves the ileum, cecum, and ascending and Infants and children with intussusception present with
transverse colon. a variety of signs and symptoms. Emesis, abdominal pain,
bloody stool, and lethargy are among the most common.
Discussion Unfortunately, these symptoms are nonspecific and carry
Intussusception occurs when a portion of the proximal a vast differential diagnosis. The classic currant jelly stool
segment of the bowel (the intussusceptum) invaginates into is generally a late finding that occurs when the bowel wall
a distal bowel segment (the intussuscipiens). Without treat- undergoes significant vascular injury and blood mixes
ment, bowel obstruction, necrosis, perforation, and death with mucoid material to produce this characteristic ap-
can ensue. Fortunately, most intussusceptions can be re- pearance. Either gross or occult blood in the stool is present
duced by contrast enema. However, some patients require in more than half of children with confirmed intussuscep-
surgical intervention with manual reduction or excision of tion. The classic triad of abdominal pain, vomiting, and
the affected portion of the bowel. Longer duration of intus- bloody stool is observed in less than 50% of affected in-
susception is associated with increased morbidity and mortal- dividuals and should not be relied on for diagnosis. How-
ity. Thus, prompt recognition and treatment are imperative. ever, this triad is highly predictive of intussusception. To
Diagnosis is contingent on clinical suspicion and confirma- date, no reliable individual discriminating factors have
tion by abdominal ultrasonography or contrast enema. been identified, and the diagnosis of intussusception con-
Intussusception is the most common cause of bowel ob- tinues to rely on a high degree of clinical suspicion and
struction in children between age 6 and 36 months. Infants confirmatory imaging studies.

e48 Pediatrics in Review Vol.34 No.12 December 2013


visual diagnosis

Abdominal radiography, ultrasonography, and contrast


enema are the 3 most common imaging modalities used
in the evaluation of a child with possible intussusception.
Abdominal radiography is occasionally indicative of intus-
susception, but the sensitivity of radiography is too low
(29%-50%) to safely exclude the diagnosis. When intussus-
ception is suspected, it must be definitely ruled out with
either ultrasonography or contrast enema. The sensitivity
and specificity of both studies approach 100%. However,
ultrasonography is the preferred initial test because it is
not invasive and does not expose the patient to ionizing
radiation. If intussusception is found on ultrasonography,
stable patients should proceed to contrast enema under
ultrasonic or fluoroscopic guidance for initial therapy. Un-
stable patients and those with evidence of bowel rupture
should have emergent surgical consultation. Figure 4. Necrotic ileum within the intussuscepted colon.

Differential Diagnosis Most pediatric surgeons approach uncomplicated, irreduc-


The differential diagnosis depends on the presenting symp- ible intussusception with laparoscopic reduction. If neces-
toms. Gastrointestinal bleeding and abdominal pain in sary, manual reduction of the intussusception is often
children can be caused by Henoch-Schönlein purpura, he- successful; however, bowel excision may also be necessary.
molytic uremic syndrome, and allergic colitis. Vomiting and A delay in definitive diagnosis may lead to greater bowel
abdominal pain have an equally broad differential diagnosis, ischemia and greater likelihood of resection.
including infectious gastroenteritis and malrotation with
volvulus. From a surgical standpoint, the presence of hem- Patient Course
atochezia, a palpable abdominal mass, and a curvilinear Unfortunately, the patient’s intussusception could not be
mass within the course of the colon with a paucity of air in reduced by contrast enema. The patient was taken to the
the bowel on abdominal radiography suggests segmental operating room for an exploratory laparotomy and man-
bowel obstruction from either volvulus or a persistent om- ual reduction of the intussusception. During the explor-
phalomesenteric duct besides an intussusception. atory laparotomy, the patient was found to have necrotic
ileum within the intussuscipiens (Fig 4). A right hemico-
Management lectomy with primary anastomosis was performed. Two
Contrast enema continues to be the diagnostic gold stan- days later the patient had abdominal distention with free
dard and first-line therapy for an intussusception. A total air seen on abdominal radiography. She returned to the
of 46% to 80% of intussusceptions can be reduced with operating room, where an anastomotic leak was repaired.
enema. Traditionally, a liquid enema was performed un- After 3 weeks in the pediatric intensive care unit, she was
der fluoroscopic guidance; however, the current trend is discharged home in good condition.
toward air enemas under ultrasonic guidance, which do
not use ionizing radiation. The efficacy of the 2 treat- Summary
ments is similar, and the choice of technique primarily de-
pends on the experience of the attending radiologist. • Children with intussusception can present with a wide
Before any planned contrast enema, intravenous access variety of symptoms, including vomiting, fever,
should be obtained, the child should be fully fluid resus- lethargy, and abdominal pain. The classic triad of
abdominal pain, hematochezia, and palpable
citated, and a pediatric surgeon should be notified. Many
abdominal mass is seen in a few patients.
physicians will also administer a dose of intravenous antibi- • Early diagnosis of intussusception depends on a high
otics. When contrast enema reduces intussusception, the level of clinical suspicion in any child with nonspecific
child should be closely monitored because 10% of patients abdominal findings followed by appropriate
will experience intussusception again. Primary surgical treat- radiographic or ultrasonographic evaluation and
confirmation with a contrast enema.
ment is indicated for unstable patients and those with signs
• Abdominal radiography, although an appropriate
of bowel perforation. Surgical treatment is also necessary component of the initial workup for gastrointestinal
when contrast enema fails to reduce the intussusception.

Pediatrics in Review Vol.34 No.12 December 2013 e49


visual diagnosis

Suggested Reading
symptoms, lacks the sensitivity to reliably exclude the Fischer TK, Bihrmann K, Perch M, et al. Intussusception in early
presence of intussusception. childhood: a cohort study of 1.7 million children. Pediatrics.
• Because ultrasonography is a safe, sensitive, and
2004;114(3):782–785
specific test for the diagnosis of intussusceptions, it
Kaiser AD, Applegate KE, Ladd AP. Current success in the
should be performed early whenever there is clinical
treatment of intussusception in children. Surgery. 2007;142
suspicion of intussusception.
(4):469–475.
• Contrast enema is the gold standard for diagnosis and
Mandeville K, Chien M, Willyerd FA, Mandell G, Hostetler MA,
first-line treatment of intussusception. There is an
increasing trend for pneumatic reduction of Bulloch B. Intussusception: clinical presentations and imag-
intussusception compared with hydrostatic reduction. ing characteristics. Pediatr Emerg Care. 2012;28(9):842–
Intravenous placement, fluid resuscitation, and 844
notification of the pediatric surgeon should be Samad L, Marven S, El Bashir H, et al. Prospective surveillance
completed before contrast enema. study of the management of intussusception in UK and Irish
infants. Br J Surg. 2012;99(3):411–415

e50 Pediatrics in Review Vol.34 No.12 December 2013

Das könnte Ihnen auch gefallen